Iggy: Chapter 21 Chapter 22 MedSurg Exam 4 Review (Evolve Questions)

Lakukan tugas rumah & ujian kamu dengan baik sekarang menggunakan Quizwiz!

The nurse is teaching a client with a neurogenic bladder to use intermittent self-catheterization for bladder emptying. Which client statement indicates a need for further clarification? "Proper handwashing before I start the procedure is very important." "My family members can be taught to help me if I need it." "A small-lumen catheter will help prevent injury to my urethra." "I will use a new, sterile catheter each time I do the procedure."

"I will use a new, sterile catheter each time I do the procedure."

Which statement by a client who has systemic lupus erythematosus (SLE) indicates to the nurse that more education about the disorder and its management is needed? "My friend and I are going to start walking 2 miles daily." "Taking my temperature every day can help me recognize when a flair is starting." "If I still have a lot of pain after taking an NSAID, I can also take acetaminophen." "At the first sign of a flare, I will begin taking my medication again."

"At the first sign of a flare, I will begin taking my medication again." The client's statement suggests that he or she believes that daily medication is not needed and would be required only during a flare. However, daily drug therapy is essential to slow the progression of the disease and organ damage.Low-impact exercise such as walking is highly recommended to maintain mobility and promote cardiovascular health. Fevers are often associated with a flare. There is no contraindication to taking both NSAIDs and acetaminophen.

A male client being treated for bladder cancer has a live virus compound instilled into his bladder as a treatment. What instruction does the nurse provide for post procedure home care? "Underwear worn during the procedure and for 12 hours afterward should be discarded." "Please be sure to stand when you are urinating." "After 12 hours, your toilet should be cleaned with a 10% solution of bleach." "Do not share your toilet with family members for the next 24 hours."

"Do not share your toilet with family members for the next 24 hours."

The nurse is assessing a young adult client who missed multiple work days this winter due to having pneumonia or other respiratory infection four times. What question would be most appropriate for the nurse to ask as part of the health interview? "Have you received your pneumonia vaccines?" "Do you have any environmental concerns at work?" "Did you have the flu before developing pneumonia?" "Do you travel out of the country a lot?"

"Do you have any environmental concerns at work?" The client may be exposed to inanimate substances in the work environment, such as mold, toxic metals, or asbestos. This particulate matter exposure can cause respiratory infections and allergies. Traveling can also predispose a client to infections, but this factor is less likely to be a major risk factor. Pneumonia vaccines are usually given for clients who are over 65 years of age. Having influenza can lead to pneumonia is the client has a depressed immune system or does not take care of him- or herself.

What response by the nurse would be most therapeutic when a client who has systemic lupus erythematosus (SLE) says, "My face has changed so much. I feel really ugly"? "I know what you mean, I feel that way sometimes too." "I bet that was hard to say. Thank you for trusting me with your feelings." "Don't worry, treatment will make everything better." "You look great. It's what is inside that counts."

"I bet that was hard to say. Thank you for trusting me with your feelings." "I bet that was hard to say. Thank you for trusting me with your feelings" is an empathetic response in a hard conversation. It acknowledges the client's bravery for sharing and encourages further therapeutic communication."You look great. It's what is inside that counts" is dismissive of the client's feelings. "Don't worry we will make everything better" is considered false reassurance, this can discount the client's feelings. "I know what you mean, I feel that way sometimes too" is focused on the nurse at a time when the focus should be on the client. All three responses hinder a continued conversation and therapeutic communication.

The nurse educates a group of women who have had frequent urinary tract infections (UTIs) about how to avoid recurrences. Which client statement shows understanding of the teaching? "Trying to get to the bathroom to urinate every 6 hours is important for me." "Urinating 1000 mL on a daily basis is a good amount for me." "I need to be drinking at least 1.5 to 2.5 L of fluids every day." "It is a good idea for me to reduce germs by taking a tub bath daily."

"I need to be drinking at least 1.5 to 2.5 L of fluids every day."

The nurse is instructing an older adult female client about interventions to decrease the risk for cystitis. Which client statement indicates that the teaching was effective? "I need to douche vaginally once a week." "I will not drink fluids after 8 p.m. each evening." "I need to drink 2½ L of fluid every day." "I must avoid drinking carbonated beverages."

"I need to drink 2½ L of fluid every day."

The nurse is questioning a female client with a urinary tract infection (UTI) about her antibiotic drug regimen. Which client statement requires further teaching? "I try to drink 3 L of fluid a day." "I take my medication when I have symptoms." "I don't use bubble baths." "I wipe front to back."

"I take my medication when I have symptoms."

A client is admitted for extracorporeal shock wave lithotripsy (ESWL). What information obtained on admission is most critical for a nurse to report to the primary health care provider before the ESWL procedure begins? "I have been taking cephalexin for an infection." "I previously had several ESWL procedures performed." "Blood in my urine has decreased, so maybe I don't need this procedure." "I take over-the-counter naproxen twice a day for joint pain."

"I take over-the-counter naproxen twice a day for joint pain."

The nurse is teaching a client who is scheduled for a neobladder and a Kock pouch. Which client statement indicates a correct understanding of these procedures? "I need to wear loose-fitting pants so the urine can flow into my ostomy bag." "If I restrict my oral intake of fluids, the adjustment will be easier." "I must go to the restroom more often because my urine will be excreted through my anus." "I will have to drain my pouch with a catheter."

"I will have to drain my pouch with a catheter."

The nurse is teaching the importance of a low purine diet to a client admitted with urolithiasis consisting of uric acid. Which client statement indicates that teaching was effective? "I will quit growing rhubarb in my garden since I'm not supposed to eat it anymore." "I will no longer be able to have red wine with my dinner." "I am so relieved that I can continue eating my fried fish meals every week." "My wife will be happy to know that I can keep enjoying her liver and onions recipe."

"I will no longer be able to have red wine with my dinner."

Which client statement about the use and care of an epinephrine autoinjector for a peanut allergy indicates to the nurse that more teaching is needed? "If I inject myself, I will still go immediately to the emergency department." "When needed, I can inject the drug right through my clothing." "My wife and I will both practice putting the device together." "If I keep the injector in the refrigerator, the drug will not expire as quickly."

"If I keep the injector in the refrigerator, the drug will not expire as quickly." Although it is true that the drug may not deteriorate as quickly if refrigerated, the client needs to have the drug with him or her at all times to use as soon as symptoms of anaphylaxis occur in order to prevent death. All other statements for the use and care of an epinephrine autoinjector are correct.

What is the nurse's best response to a client newly diagnosed with systemic lupus erythematosus (SLE) who asks why nicotine use, especially cigarette smoking or vaping, should be avoided? "Nicotine reduces blood flow to your organs and increases the risk for permanent damage." "Using nicotine in any form reduces the effectiveness of drug therapy for lupus." "Nicotine promotes muscle cell loss, increases joint inflammation, and reduces functional mobility." "Smoking or vaping increases your risk for lung cancer development."

"Nicotine reduces blood flow to your organs and increases the risk for permanent damage." Nicotine in any form constricts blood vessels and reduces perfusion. Perfusion is already reduced by the vasculitis that is part of the disease. Thus, use of nicotine greatly increases the risk for necrosis of many tissues and organs. Although smoking or vaping do increase the risk for lung cancer, their effects on blood vessels are a greater issue for the client with SLE. Nicotine neither reduces the effectiveness of drug therapy nor promotes muscle cell loss.

The nurse is caring for a client who has methicillin-resistant staphylococcus aureus (MRSA) infection and is starting oral delafloxacin therapy. What health teaching would the nurse include about this drug? "Take the drug every day until you feel you better or until your fever does away." "Take the drug at least 2 hours before or 6 hours after any antacids or minerals." "Take the drug every other day as prescribed unless you feel nauseated." "If you forget a dose of the drug, wait until the next day to take the next dose."

"Take the drug at least 2 hours before or 6 hours after any antacids or minerals." Delafloxacin interacts with metals such as magnesium and iron. Therefore, the drug must not be given when drugs containing metals are in the stomach.

An older adult woman confides to the nurse, "I am so embarrassed about buying adult diapers for myself." How does the nurse respond? "That is tough. What do you think might help?" "Tell everyone that they are for your husband." "Shop at night, when stores are less crowded." "Don't worry about it. You need them."

"That is tough. What do you think might help?"

What is the nurse's best response to a client who had a severe allergic reaction to shrimp states, "I have had shrimp once before and did not have a reaction. Why is this happening now?" "Allergies are tricky, and many reasons for responses are not known." "It is most likely that you didn't eat enough shrimp the first time to cause a reaction." "The first time your body recognized the shrimp as an allergen, and the second time it reacted to it." "This means you may be allergic to something else and not to shrimp."

"The first time your body recognized the shrimp as an allergen, and the second time it reacted to it." Type I reactions have two parts. During the first exposure, the client makes antigen-specific IgE, and becomes sensitized to the allergen. When the sensitized client is re-exposed to the allergen, a more severe reaction occurs.To point out the amount of shrimp eaten is not helpful and could make the client believe that eating only a small amount of shrimp would not cause a reaction. The same is true for option C. Stating that allergies "are tricky" does not help to inform or educate the client about what he or she should do to prevent harm. This response may make the client afraid of everything in his or her environment.

The nurse is teaching a client about pelvic muscle exercises. What information does the nurse include? "For the best effect, perform all of your exercises while you are seated on the toilet." "You are exercising correct muscles if you can stop urine flow in midstream." "Limit your exercises to 5 minutes twice a day, or you may injure yourself." "Results should be visible to you within 72 hours."

"You are exercising correct muscles if you can stop urine flow in midstream."

The nurse is caring for a client with chest pain. What assessment data would cause the nurse to suspect unstable angina? Select all that apply. A. ST changes B. Troponin T 0.6 ng/mL C. Pain lasts 15-25 minutes D. Increased number of angina attacks E. The intensity of the chest pain has increased.

A. ST changes C. Pain lasts 15-25 minutes D. Increased number of angina attacks E. The intensity of the chest pain has increased.

A client with a urinary tract infection is prescribed trimethoprim/sulfamethoxazole (Bactrim). What information does the nurse provide to this client about taking this drug? (Select all that apply.) "You will need to take all of these drugs to get the benefits." "Drink at least 3 L of fluids every day." "Be certain to wear sunscreen and protective clothing." "Take this drug with 8 ounces (236 mL) of water." "Try to urinate frequently to keep your bladder empty."

"You will need to take all of these drugs to get the benefits." "Drink at least 3 L of fluids every day." "Be certain to wear sunscreen and protective clothing." "Take this drug with 8 ounces (236 mL) of water."

A client with acute kidney injury is receiving a fluid challenge of 500 mL of normal saline over 1 hour. With a drop factor of 20 drops/mL, how many drops per minute does the nurse infuse? _________

1.67 DROPS/MIN

The nurse receives the change-of-shift report on four clients. Which client does the nurse decide to assess first? A 26 year old admitted 2 days ago with urosepsis with an oral temperature of 99.4° F (37.4° C). A 32 year old admitted with hematuria and possible bladder cancer who is scheduled for cystoscopy. A 40 year old with noninfectious urethritis who is reporting "burning" and has estrogen cream prescribed. A 28 year old with urolithiasis who has been receiving morphine sulfate and has not voided for 8 hours.

A 28 year old with urolithiasis who has been receiving morphine sulfate and has not voided for 8 hours.

Which client with persistent joint and muscle pain will the nurse consider as most likely to have a systemic lupus erythematosus (SLE) diagnosis? A 33-year-old African-American man whose father died from a myocardial infarction. A 33-year-old white woman whose sister has Grave disease. A 33-year-old African-American woman whose mother has psoriasis. A 33-year-old man whose identical twin brother has acute myelogenous leukemia.

A 33-year-old African-American woman whose mother has psoriasis. SLE is an autoimmune disorder that is much more common in women than in men and has a genetic predisposition related to tissue type. A client with SLE is very likely to have another close relative who also has an autoimmune disorder, such as psoriasis (myocardial infarction, type 2 diabetes mellitus, and thrombotic stroke are not autoimmune disorders). In addition, the incidence of SLE is about eight times greater for African-American women than for white women.

For which hypersensitivity situation will the nurse prepare a client for management with plasmapheresis? A 35 year old with drug-induced hemolytic anemia A 30 year old with poison ivy lesions on 60% of the body A 25 year old with penicillin-induced anaphylaxis A 40 year old with angioedema and tongue swelling

A 35 year old with drug-induced hemolytic anemia Drug-induced hemolytic anemia is a type II hypersensitivity reaction in which the body makes autoantibodies directed against red blood cells that have foreign proteins from the drug attached to them. In this type of reaction, the autoantibody binds to red blood cells, forming immune complexes that destroy red blood cells along with the attached protein. Management starts with discontinuing the offending drug and, performing plasmapheresis (filtration of the plasma to remove specific substances) to remove the formed autoantibodies. Plasmapheresis is not beneficial with other types of hypersensitivity reactions.

Which client does the nurse manager on the medical unit assign to an experienced LPN/LVN? A 46 year old scheduled for cystectomy who needs help in selecting a stoma site. A 48 year old receiving intravesical chemotherapy for bladder cancer. A 55 year old with incontinence who has intermittent catheterization prescribed. A 42 year old with painless hematuria who needs an admission assessment.

A 55 year old with incontinence who has intermittent catheterization prescribed.

Which client is at greatest risk for developing an infection? A 65-year-old woman who had heart surgery 4 days ago A 54-year-old man with hypertension A 21-year-old woman with a fractured tibia in a cast A 71-year-old man in a nursing home

A 65-year-old woman who had heart surgery 4 days ago Older clients such as the 65-year-old woman with compromised skin (surgical incision) are at the highest risk for infection.No coexisting conditions are present for the client with hypertension to be at risk for infection. The 71-year-old client in a nursing home is not at highest risk because no coexisting conditions make this client most vulnerable to infection.

What type of health problem will the nurse expect to see in a client who has very few regulator T cells? A)Increased severity of allergic and other hypersensitivity reactions B)Decreased ability to recognize non-self cells C)Decreased immunoglobulin production D)Increased risk for cancer development

A) Increased severity of allergic and other hypersensitivity reactions Regulator T-cells (Tregs) function to limit the actions of general and specific responses. These cells prevent over-responses to the presence of "foreign proteins" within a person's environment. People who are deficient in these cells have more severe hypersensitivity reactions, allergies, and autoimmune responses.

With which client conditions will the nurse expect an inflammatory response without infection? (Select all that apply.) A) Poison ivy rash B) Otitis media C) Welt formation after a bee sting D) Blister formation from a burn E) Blister from a cold sore F) Acute myocardial infarction

A) Poison ivy rash B) Welt formation after a bee sting C) Blister formation from a burn E) Acute myocardial infarction Inflammation is nonspecific response to tissue injury, irritation, and invasion by organisms or allergens. Options A, B, C, and E have tissue injury, irritation, or invasion as the pathophysiologic mechanism causing the response. Otitis media is an inflammation occurring with a bacterial or viral infection. Blister formation with a cold sore occurs as in response to the presence of the Herpes simplex virus and is highly infectious.

Which cancer screening or prevention activity is most important for the nurse to include when assessing a 20-year-old client who has Down syndrome? A. Assessing his skin for bruises and petechaie B. Teaching him how to perform self-testicular examination C. Testing his stool for occult blood D. Encouraging him to eat more fruit and leafy, green vegetables

A. Assessing his skin for bruises and petechaie Rationale:All of the screening and prevention activities are appropriate; however, people with Down syndrome have an increased life-time risk for the development of leukemia.

Which precautions are most important for the nurse to teach as part of health promotion for inflammation and immunity to an 88-year-old client? (Select all that apply.) A)Report any temperature elevation to your primary health care provider immediately. B)Get an influenza vaccination every year. C)Wear gloves when working in your garden. D)Avoid performing any level of aerobic exercise. E)Consider moving into an assisted living facility. F)Be sure to have a tuberculosis skin test every year.

A)Report any temperature elevation to your primary health care provider immediately. B)Get an influenza vaccination every year. C)Wear gloves when working in your garden. Older clients have overall reduced immunity and a higher risk for developing influenza and any other respiratory tract infection. They should receive annual influenza vaccinations. The skin of older adults is thinner, drier, and a greater risk for injury and infection. Wearing gloves when gardening can help prevent injury and reduce the risk for infection. Older clients often do not have greatly elevated temperatures during infection, which contributes to the infection being overlooked until it becomes serious. Thus, older clients should report any increase in temperature above their normal range to identify infections at earlier stages. TB skin tests may be falsely negative in older clients with reduced immunity and annual testing is of no real benefit. Healthy older clients who are cognitively intact and able to care for themselves have no need to change their living arrangements unless they so desire. Older clients can still engage in low-impact aerobic exercise under the supervision of their primary health care provider.

Which types of problems will the nurse expect to find more frequently in a client who does not make adequate amounts of immunoglobulin A (IgA)? (Select all that apply.) A)Upper respiratory infections B)Cystitis C)Excessive bleeding D)Contact dermatitis E)Anaphylaxis F)Diarrhea

A)Upper respiratory infections B)Cystitis F)Diarrhea IgA is the secretory immunoglobulin that is present in highest concentrations in the secretions of the mouth, gastrointestinal tract, and genitourinary tract. IgA helps prevent infections in these body areas and does not circulate in significant amounts. It is not associated with any types of allergic reactions such as anaphylaxis or contact dermatitis. It plays no role in the blood clotting cascade.

A client with angina has received education about acute coronary syndrome. Which client statement indicates understanding? A. "This is a warning sign and I need to change my lifestyle to prevent a heart attack." B. "Angina is a temporary blood flow problem that will resolve." C. "I need to tell my wife I've had a heart attack." D. "Because this is temporary, I don't need medications for my heart."

A. "This is a warning sign and I need to change my lifestyle to prevent a heart attack."

1. The nurse is teaching a client with heart failure about a newly prescribed medication, ivabradine. What teaching will the nurse include? Select all that apply. A. "Visual changes with exposure to light are expected initially." B. "Be sure to take this medication with food." C. "Call your healthcare provider if your pulse rate is low or irregular." D. "Use caution when driving in the sunlight." E. "Check your BP regularly and notify the healthcare provider if elevated."

A. "Visual changes with exposure to light are expected initially." B. "Be sure to take this medication with food." C. "Call your healthcare provider if your pulse rate is low or irregular." D. "Use caution when driving in the sunlight." E. "Check your BP regularly and notify the healthcare provider if elevated." (Yes, they are all correct- new NCLEX format) Rationale: Ivabradine is an HCN channel blocker that slows the heart rate. Side effects include: bradycardia, hypertension, atrial fibrillation, and luminous phenomena (visual brightness) The nurse will teach the client that visual changes are expected initially. The nurse will advise to take this medication with meals and teach the client how to check radial pulse and to report low heart rate or irregularity to the health care provider. The nurse will also teach clients that visual changes are associated with light and clients should use caution when driving or using machines in situations where light intensity may change abruptly.

The nurse is teaching a class on the management of sepsis. What teaching will the nurse include regarding the Hour-1 sepsis management bundle? (Select all that apply.) A. A bundle is a group of two or more interventions that has been shown to be effective when applied in a sequence. B. Initiate insulin therapy according to blood glucose levels. C. Measure fibrinogen levels. D. Measure lactate levels. E. Begin rapid administration of crystalloids for hypotension. F. Administer broad spectrum antibiotics.

A. A bundle is a group of two or more interventions that has been shown to be effective when applied in a sequence. D. Measure lactate levels. E. Begin rapid administration of crystalloids for hypotension. F. Administer broad spectrum antibiotics.

Which client has a risk for hypovolemic shock? A. A client with esophageal varices B. A client with kidney failure C. A client with arthritis taking daily acetaminophen D. A client with pain from a kidney stone

A. A client with esophageal varices

Which finding in the first 24 hours after kidney transplantation requires immediate intervention? A. Abrupt decrease in urine output B. Blood-tinged urine C. Incisional pain D. Increase in urine output

A. Abrupt decrease in urine output

The nurse caring for a client with heart failure who is taking digoxin. What assessment data requires that nurse notify the health care provider? (Select all that apply.) A. Anorexia B. Blurred vision C. Fatigue D. Heart rate 110/beats/min E. Serum digoxin level of 1.5 ng/mL (1.92 nmol/L)

A. Anorexia B. Blurred vision C. Fatigue Rationale: - The signs and symptoms of digoxin toxicity that the nurse notifies the provider include: blurred vision, fatigue, and anorexia. Changes in mental status, especially in older adults, may also occur.- Sinus bradycardia and not tachycardia is a sign of digoxin toxicity.- A serum digoxin level between 0.5 and 2.0 (1.02 and 2.56 nmol/L) is considered normal and is not a symptom.

A client is exhibiting signs and symptoms of early shock. Which nursing actions support the psychosocial integrity of the client? (Select all that apply.) A. Ask family members to stay with the client. B. Call the health care provider. C. Increase IV and oxygen rates. D. Remain with the client. E. Reassure the client that everything is being done for him or her.

A. Ask family members to stay with the client. D. Remain with the client. E. Reassure the client that everything is being done for him or her.

A client with chronic kidney disease reports chest pain. The nurse notes tachycardia and low-grade fever. Which additional assessment is warranted? A. Auscultate for pericardial friction rub. B. Assess for crackles. C. Monitor for decreased peripheral pulses. D. Determine if the client is able to ambulate.

A. Auscultate for pericardial friction rub.

The home health nurse visits a client with heart failure who has gained 5 lb (2.3 kg) in the past 3 days. The client states, "I feel so tired and short of breath." Which action does the nurse take first? A. Auscultate the client's posterior breath sounds. B. Notify the health care provider about the client's weight gain. C. Remind the client about dietary sodium restrictions. D. Assess the client for peripheral edema.

A. Auscultate the client's posterior breath sounds. Rationale: Because the client is at risk for pulmonary edema and hypoxemia, the first action should be to assess breath sounds. Assessment of edema may be delayed until after breath sounds are assessed. After a full assessment, the nurse should notify the health care provider. After physiologic stability is attained, then ask the client about behaviors that may have caused the weight gain, such as increased sodium intake or changes in medications.

A client with a recently created vascular access for hemodialysis is being discharged. In planning discharge instructions, which information does the nurse include? A. Avoiding venipuncture and blood pressure measurements in the affected arm B. Modifications to allow for complete rest of the affected arm C. How to assess for a bruit in the affected arm D. How to practice proper nutrition

A. Avoiding venipuncture and blood pressure measurements in the affected arm

The nurse is assessing a client with septic shock. What assessment data indicates a progression of shock? Select all that apply. A. BP change from 86/50 to 100/64 B. HR change from 98 to 76 C. Cool and clammy skin D. Petechiae along the gum line E. Urine output 45 ml/hr

A. BP change from 86/50 to 100/64 C. Cool and clammy skin D. Petechiae along the gum line

The nurse is caring for a client with heart failure in a cardiac clinic. What assessment data indicates that the client has demonstrated a positive outcome related to the addition of metoprolol to the medication regimen? A. Client states, "I can sleep on one pillow." B. Current ejection fraction is 25%. C. Client reports feeling like her heart beats very fast at times. D. Records indicate five episodes of pulmonary edema last year.

A. Client states, "I can sleep on one pillow." Rationale: Improvement in activity tolerance, less orthopnea, and improved symptoms represent a positive response to beta blockers. An ejection fraction of 25% is well below the normal of 50% to 70% and indicates poor cardiac output. Repeated hospitalization for acute exacerbation of left-sided heart failure does not demonstrate a positive outcome. Although metoprolol decreases the heart rate, palpitations are defined as the feeling of the heart beating fast in the chest; this is not a positive outcome.

The RN has just received change-of-shift report. Which of the assigned clients should be assessed first? A. Client with chronic kidney failure who was just admitted with shortness of breath B. Client with kidney insufficiency who is scheduled to have an arteriovenous fistula inserted C. Client with azotemia whose blood urea nitrogen and creatinine are increasing D. Client receiving peritoneal dialysis who needs help changing the dialysate bag

A. Client with chronic kidney failure who was just admitted with shortness of breath

The assistive personnel (AP) is concerned about a postoperative client with blood pressure (BP) of 90/60 mm Hg, heart rate of 80 beats/min, and respirations of 22 breaths/min. What is the appropriate nursing action? A. Compare these vital signs with the last several readings. B. Request that the surgeon see the client. C. Increase the rate of intravenous fluids. D. Reassess vital signs using different equipment.

A. Compare these vital signs with the last several readings.

To prevent pre-renal acute kidney injury, which person is encouraged to increase fluid consumption? A. Construction worker B. Office secretary C. Schoolteacher D. Taxicab driver

A. Construction worker

1. The nurse is admitting an 84-year-old client with heart failure to the emergency department with confusion, blurry vision, and an upset stomach. Which assessment data is most concerning? A. Digoxin therapy daily. B. Daily metoprolol. C. Furosemide twice daily. D. Currently taking an antacid for upset stomach.

A. Digoxin therapy daily. Rationale: Confusion, blurry vision, and upset stomach are symptoms of Digoxin toxicity, which is common in older adults and requires immediate treatment. The other answers are important assessment data but do not indicate immediate connection to the client's presentation.

When assisting with dietary protein needs for a client on peritoneal dialysis, the nurse recommends that the client select which food? A. Eggs B. Ham C. Eggplant D. Macaroni

A. Eggs

The nurse is caring for a client with heart failure. What assessment data will the nurse anticipate? (Select all that apply.) A. Fatigue B. Sleeping on back without a pillow C. Chest discomfort or pain D. Tachycardia E. Expectorating thick, yellow sputum

A. Fatigue C. Chest discomfort or pain D. Tachycardia Rationale: When caring for a client with heart failure, the nurse needs to assess for chest discomfort or pain, tachycardia, and fatigue. - Decreased tissue perfusion with heart failure may cause chest pain or angina. - Tachycardia may occur as compensation for or as a result of decreased cardiac output. - Fatigue is a symptom of poor tissue perfusion in clients with heart failure.- Presence of a cough or dyspnea results as pulmonary venous congestion ensues. - Clients with acute heart failure have dry cough and, when severe, pink, frothy sputum. Thick, yellow sputum is indicative of infection. - Position for sleeping isn't a symptom. Clients usually find it difficult to lie flat because of dyspnea symptoms.

Which clients are at risk for acute kidney injury (AKI)? (Select all that apply.) A. Football player in preseason practice B. Client who underwent contrast dye radiology C. Accident victim recovering from a severe hemorrhage D. Accountant with diabetes E. Client in the intensive care unit on high doses of antibiotics F. Client recovering from gastrointestinal influenza

A. Football player in preseason practice B. Client who underwent contrast dye radiology C. Accident victim recovering from a severe hemorrhage E. Client in the intensive care unit on high doses of antibiotics F. Client recovering from gastrointestinal influenza

The nurse is caring for a patient in the initial stage of hypovolemic shock. What assessment data will the nurse anticipate? A. Heart rate 118 B. 2+ pedal pulses C. Bilateral fine crackles in lung bases D. BP change from 100/60 to 100/40

A. Heart rate 118

The nurse is assessing a client who had a coronary artery bypass graft yesterday. Which assessment data indicates the client is at risk for decreased perfusion? A. Heart rate of 50 beats/min B. Potassium level of 4.2 mEq/L C. Systolic blood pressure of 120 mm/Hg D. 50 ml of bloody drainage in chest tube over 4 hours

A. Heart rate of 50 beats/min

Which conditions does the nurse teach a client are some of the seven warning signs of cancer? (Select all that apply.) A. Heavy nosebleeds independent of trauma to the nasal mucosa B. Menstrual bleeding that has decreased C. Increased pigmentation with deeper coloring in a mole D. Difficulty starting the stream of urine for the past 6 months E. Indigestion regardless of food type eatenF. Thickening of breast tissue in one area

A. Heavy nosebleeds independent of trauma to the nasal mucosa C. Increased pigmentation with deeper coloring in a mole D. Difficulty starting the stream of urine for the past 6 months E. Indigestion regardless of food type eaten F. Thickening of breast tissue in one area Rationale:The seven warning signs of cancer include persistent changes in bladder habits, unusual bleeding without trauma, obvious change in a wart or mole, chronic or persistent indigestion (especially if not associated with any food type), and the presence of a lump or thickening (often in the breast but can be anywhere). Reduced menstrual flow is not associated with a malignancy.

Which clinical manifestation indicates the need for increased fluids in a client with kidney failure? A. Increased blood urea nitrogen (BUN) B. Increased creatinine level C. Pale-colored urine D. Decreased sodium level

A. Increased blood urea nitrogen (BUN)

A client in the progressive stage of hypovolemic shock has all of the following signs, symptoms, or changes. Which signs will the nurse attribute to ongoing compensatory mechanisms? Select all that apply. A. Increasing pallor B. Increasing thirst C. Increasing confusion D. Increasing heart rate E. Increasing respiratory rate F. Decreasing systolic blood pressure G. Decreasing blood pH H. Decreasing urine output

A. Increasing pallor B. Increasing thirst D. Increasing heart rate E. Increasing respiratory rate H. Decreasing urine output

Which common cancers will the nurse inform clients are related to tobacco use? (Select all that apply.) A. Lung cancer B. Cancer of the larynx C. Bladder cancer D. Cancer of the tongue E. Skin cancer F. Cardiac cancer

A. Lung cancer B. Cancer of the larynx C. Bladder cancer D. Cancer of the tongue Rationale:Organs exposed to the carcinogens in tobacco (lungs, tongue, larynx) are most likely to develop cancer. Bladder cancer is also associated with cigarette smoking because many of the carcinogens in tobacco are filtered into the urine and come into contact with the urinary bladder. Oral cancer is also a risk with "smokeless" tobacco. The heart does not contain cells that divide; therefore, cardiac cancer is unlikely. Skin cancer generally is related to repeated sun and other ultraviolet exposure, such as that found with tanning beds.

Which client assessment findings indicate to the nurse that leukemia may be present? (Select all that apply.) A. Multiple bruises B. Night sweats C. Severe epistaxis D. Fever E. Frequent colds F. Fatigue

A. Multiple bruises B. Night sweats C. Severe epistaxis D. Fever E. Frequent colds F. Fatigue Rationale:All of the answers can be linked to leukemia, especially when they occur together. (Other issues can account for any one of them when they occur singly). Leukemia is a blood and bone marrow cancer. Prolonged bleeding (bruises and epistaxis) can be caused by immature white cells crowding the client's platelets. Night sweats are often caused by fevers that are common with leukemia. Fatigue can be caused by the presence of persistent infection or by the cancer itself as it grows. Decreased ability to fight infection (frequent colds) is caused by the lack of mature white blood cells, as leukemic cells cannot function properly. Fever is associated with an increased rate of metabolism among the leukemic cells and the presence of any infection.

Which cancer type does the nurse interpret from a client's pathology report that indicates "stage 2 rhabdomyosarcoma"? A. Muscle B. Brain C. Bone D. Breast

A. Muscle

The nurse teaches a client who is recovering from acute kidney disease to avoid which type of medication? A. Nonsteroidal anti-inflammatory drugs (NSAIDs) B. Angiotensin-converting enzyme (ACE) inhibitors C. Opiates D. Calcium channel blockers

A. Nonsteroidal anti-inflammatory drugs (NSAIDs)

Which nursing action may be delegated to assistive personnel (AP) working on the medical unit? A. Obtain daily weights for several clients with class IV heart failure. B. Check for peripheral edema in a client with endocarditis. C. Monitor the pain level for a client with acute pericarditis. D. Determine the usual alcohol intake for a client with cardiomyopathy.

A. Obtain daily weights for several clients with class IV heart failure.

What effect does a "passenger" mutation in a gene have on cancer development? A. Passenger mutations do not affect cancer development but can serve as targets for specific cancer therapies. B. These mutations enhance the effectiveness of carcinogens causing direct DNA damage of a normal cell, increasing the likelihood of cancer development. C. These mutations protect against cancer development by reversing the effects of initiation. D. Passenger mutations are another term for proto-oncogene gene mutations.

A. Passenger mutations do not affect cancer development but can serve as targets for specific cancer therapies. Rationale:Although passenger mutations are often found along with driver mutations in later cancer stages, they appear to have no effect on initial cancer development or cancer cell survival. Their presence can be used to identify advanced cancer types and may also be used as "targets" for newer cancer therapies.

Which warning signs of cancer would the nurse specifically teach in a wellness course directed to a group of older adults? (Select all that apply.) A. Persistent hoarseness B. Severe heartburn C. Chronic diarrhea D. Loss of skin turgor E. Curd-like vaginal discharge F. Difficulty swallowing with meals

A. Persistent hoarseness B. Severe heartburn C. Chronic diarrhea F. Difficulty swallowing with meals Rationale:Change in bowel habits, persistent hoarseness, indigestion or difficulty swallowing are all potential warning signs of cancer. A curd-like vaginal discharge represents a yeast infection. Loss of skin turgor is a normal response to aging.

Which intervention best assists the client with acute pulmonary edema in reducing anxiety and dyspnea? A. Place the client in high-Fowler position with the legs down. B. Reassure the client that distress can be relieved with proper intervention. C. Ask a family member to remain with the client. D. Monitor pulse oximetry and cardiac rate and rhythm.

A. Place the client in high-Fowler position with the legs down. Rationale: High-Fowler's position and placing the legs in a dependent position will decrease venous return to the heart, thus decreasing pulmonary venous congestion.

The nurse assists a client with acute kidney injury (AKI) to modify the diet in which ways? (Select all that apply.) A. Restricted protein B. Liberal sodium C. Restricted fluids D. Low potassium E. Low fat

A. Restricted protein C. Restricted fluids D. Low potassium

After receiving change-of-shift report in the coronary care unit, which client will the nurse assess first? A. The client with acute coronary syndrome who has a 3-lb (1.4-kg) weight gain and dyspnea. B. The client who had a percutaneous coronary angioplasty who has a dose of heparin scheduled. C. The client who had bradycardia after a myocardial infarction and now has a paced heart rate of 64 beats/min. D. A client who has first-degree heart block, rate 68 beats/min, after having an inferior myocardial infarction.

A. The client with acute coronary syndrome who has a 3-lb (1.4-kg) weight gain and dyspnea.

A client admitted for heart failure has a priority problem of hypervolemia related to compromised regulatory mechanisms. Which nursing assessment data, obtained the day after admission, is the best indicator that the treatment has been effective? A. The client's weight decreases by 2.5 kg. B. The client has diuresis of 400 mL in 24 hours. C. The client's blood pressure is 122/84 mm Hg. D. The client has an apical pulse of 82 beats/min.

A. The client's weight decreases by 2.5 kg. Rationale: The best indicator of fluid volume gain or loss is daily weight; because each kilogram represents approximately 1 liter, this client has lost approximately 2500 mL of fluid. Diuresis of 400 mL in 24 hours represents oliguria. Although a blood pressure of 122/84 mm Hg is a normal finding, alone it is not significant for relief of hypervolemia. Although an apical pulse of 82 beats/min is a normal finding, alone it is not significant to determine whether hypervolemia is relieved.

The nurse is preparing to teach a client that metabolic syndrome can increase the risk for myocardial infarction (MI). Which signs of metabolic syndrome will the nurse include? (Select all that apply.) A. Truncal obesity B. Hypercholesterolemia C. Elevated homocysteine levels D. Glucose intolerance E. Client taking losartan

A. Truncal obesity B. Hypercholesterolemia D. Glucose intolerance E. Client taking losartan

While managing care for a client with chronic kidney disease, which actions does the registered nurse (RN) plan to delegate to unlicensed assistive personnel (UAP)? (Select all that apply.) Obtain the client's pre-hemodialysis weight. Check the arteriovenous (AV) fistula for a thrill and bruit. Document the amount the client drinks throughout the shift. Auscultate the client's lung sounds every 4 hours. Explain the components of a low-sodium diet.

AC

The oncology nurse is caring for a group of clients receiving chemotherapy. The client with which sign/symptom is displaying bone marrow suppression? A Hemoglobin of 7.4 and hematocrit of 21.8 B Potassium level of 2.9 mEq/L and diarrhea C 250,000 platelets/mm3 D 5000 white blood cells/mm3

ANS: A Bone marrow suppression causes anemia, leukopenia, and thrombocytopenia; the client with a hemoglobin of 7.4 and hematocrit of 21.8 has anemia demonstrated by low hemoglobin and hematocrit. The client with diarrhea and a potassium level of 2.9 mEq/L has hypokalemia and electrolyte imbalance. The client with 250,000 platelets/mm3 and the client with 5000 white blood cells/mm3 demonstrate normal values.

he nurse is caring for a client receiving chemotherapy who reports anorexia. Which measure does the nurse use to best monitor for cachexia? A Monitor weight B Trend red blood cells and hemoglobin and hematocrit C Monitor platelets D Observe for motor deficits

ANS: A Cachexia results in extreme body wasting and malnutrition; severe weight loss is expected. Anemia and bleeding tendencies result from bone marrow suppression secondary to invasion of bone marrow by a cancer or a side effect of chemotherapy. Motor deficits result from spinal cord compression.

The RN working on an oncology unit has just received report on these clients. Which client should be assessed first? A Client with chemotherapy-induced neutropenia who has just been admitted with an elevated temperature B Client with lymphoma who will need administration of an antiemetic before receiving chemotherapy C Client with metastatic breast cancer who is scheduled for external beam radiation in 1 hour D Client with xerostomia associated with laryngeal cancer who needs oral care before breakfast

ANS: A Neutropenia poses high risk for life-threatening sepsis and septic shock, which develop and progress rapidly in immune-suppressed people; the nurse should see the client with chemotherapy-induced neutropenia first. The client with lymphoma and the client with metastatic breast cancer are not in distress and can be assessed later. The client with dry mouth (xerostomia) can be assessed later, or the nurse can delegate mouth care to unlicensed assistive personnel.

When monitoring a client with suspected syndrome of inappropriate antidiuretic hormone (SIADH), the nurse reviews the client's medical record, which contains the following information. The nurse notifies the health care provider for which signs and symptoms consistent with this syndrome? (Select all that apply.) Physical Assessment Findings: Neuro: Episodes of confusion Cardiac: Pulse 88 and regular Musculoskeletal: Weakness, tremors Diagnostic Findings : Na: 115 K: 4.2 Creatinine: 0.8 Medications ondansetron (Zofran) cyclophosphamide (Cytoxan) A Hyponatremia B Mental status changes C Azotemia D Bradycardia E Weakness

ANS: A, B, E Antidiuretic hormone (ADH) is secreted or produced ectopically, resulting in water retention and sodium dilution. Dilutional hyponatremia results from ADH secretion, causing confusion and changes in mental status. Weakness results from hyponatremia. Azotemia refers to buildup of nitrogenous waste products in the blood, typically from renal damage. Bradycardia is not part of the constellation of symptoms related to SIADH; tachycardia may result from fluid volume excess. Awarded 0.0 points out of 1.0 possible points.

When caring for the client receiving cancer chemotherapy, which signs or symptoms related to thrombocytopenia should the nurse report to the health care provider? (Select all that apply.) A Bruises B Fever C Petechiae D Epistaxis E Pallor

ANS: A, C, D Bruising, petechiae, and epistaxis (nosebleeds) are symptoms of a low platelet count. Fever is a sign of infection secondary to neutropenia. Pallor is a sign of anemia.

Which potential side effects does the nurse include in the teaching plan for a client undergoing radiation therapy for laryngeal cancer? (Select all that apply.) A Fatigue B Changes in color of hair C Change in taste D Changes in skin of the neck E Difficulty swallowing

ANS: A, C, D, E Radiation therapy to any site produces fatigue in most clients, and may cause clients to report changes in taste. Radiation side effects are site-specific; the larynx is in the neck, so changes in the skin of the neck may occur. Dysphagia (difficulty swallowing) may occur from radiation to the throat area. Chemotherapy, which causes alopecia, may cause changes in the color or texture of hair; this does not normally occur with radiation therapy.

The nurse has received in report that a client receiving chemotherapy has severe neutropenia. Which interventions does the nurse plan to implement? (Select all that apply.) A Assess for fever. B Observe for bleeding. C Administer pegfilgrastim (Neulasta). D Do not permit fresh flowers or plants in the room. E Do not allow the client's 16-year-old son to visit. F Teach the client to omit raw fruits and vegetables from the diet.

ANS: A, C, D, F Any temperature elevation in a client with neutropenia is considered a sign of infection and should be reported immediately to the health care provider. Administration of biological response modifiers, such as filgrastim (Neupogen) and pegfilgrastim (Neulasta), is indicated in neutropenia to prevent infection and sepsis. Flowers and plants may harbor organisms such as fungi or viruses and are to be avoided for the immune-suppressed client. All fruits and vegetables should be cooked well; raw fruits and vegetables may harbor organisms. Thrombocytopenia, or low platelet levels, causes bleeding, not low neutrophils (a type of white blood cell). The client is at risk for infection, not the visitors, if they are well; however, very small children, who may get frequent colds and viral infections, may pose a risk.

10 The nurse is caring for a client who is receiving rituximab (Rituxan) for treatment of lymphoma. During the infusion, it is essential for the nurse to observe for which side effect? A Alopecia B Allergy C Fever D Chills

ANS: B Allergy is the most common side effect of monoclonal antibody therapy (rituximab). Monoclonal antibody therapy does not cause alopecia. Although fever and chills are side effects of monoclonal antibody therapy, they would not take priority over an allergic response that could potentially involve the airway.

Which manifestation of an oncologic emergency requires the nurse to contact the health care provider immediately? A New onset of fatigue B Edema of arms and hands C Dry cough D Weight gain

ANS: B Edema of the arms and hands indicates worsening compression of the superior vena cava consistent with superior vena cava syndrome. The compression must be relieved immediately, often with radiation therapy, because death can result without timely intervention. New onset of fatigue may likely be an early manifestation of hypercalcemia, which usually develops slowly, but because it is an early manifestation, this is not the priority. Dry cough is not a manifestation that is specific to an oncologic emergency; however, it may be a side effect of chemotherapy. Weight gain could be an early sign of syndrome of inappropriate antidiuretic hormone; although this should be addressed, it is an early sign so it is not the priority.

Which medication does the nurse plan to administer to a client before chemotherapy to decrease the incidence of nausea and vomiting? A Morphine B Ondansetron (Zofran) C Naloxone (Narcan) D Diazepam (Valium)

ANS: B Ondansetron is a 5-HT3 receptor blocker that blocks serotonin to prevent nausea and vomiting. Morphine is a narcotic analgesic or opiate; it may cause nausea. Naloxone is a narcotic antagonist used for opiate overdose. Diazepam, a benzodiazepine, is an antianxiety medication only; lorazepam, another benzodiazepine, may be used for nausea.

When caring for the client with hyperuricemia associated with tumor lysis syndrome (TLS), for which medication does the nurse anticipate an order? A Recombinant erythropoietin (Procrit) B Allopurinol (Zyloprim C Potassium chloride D Radioactive iodine-131 (131I)

ANS: B TLS results in hyperuricemia (elevation of uric acid in the blood), hyperkalemia, and other electrolyte imbalances; allopurinol decreases uric acid production and is indicated in TLS. Recombinant erythropoietin is used to increase red blood cell production and is not a treatment for hyperuricemia. Administering additional potassium is dangerous. Radioactive iodine-131 is indicated in the treatment of thyroid cancer, not TLS.

1. The nurse is caring for a client with end-stage ovarian cancer who needs clarification on the purpose of palliative surgery. Which outcome should the nurse teach the client is the goal of palliative surgery? A Cure of the cancer B Relief of symptoms or improved quality of life C Allowing other therapies to be more effective D Prolonging the client's survival time

ANS: B The focus of palliative surgery is to improve quality of life during the survival time. Curative surgery removes all cancer cells, visible and microscopic. Debulking is a procedure that removes some cancerous tissue, allowing other therapies to be more effective. Many therapies, such as surgery, chemotherapy, and biotherapy, increase the client's chance of cure and survival, but palliation improves quality of life.

Which client problem does the nurse set as the priority for the client experiencing chemotherapy-induced peripheral neuropathy? A Potential for lack of understanding related to side effects of chemotherapy B Potential for injury related to sensory and motor deficits C Potential for ineffective coping strategies related to loss of motor control D Altered sexual function related to erectile dysfunction

ANS: B The highest priority is safety. Although knowing the side effects of chemotherapy may be helpful, the priority is the client's safety because of the lack of sensation or innervation to the extremities. The nurse should address the client's coping only after providing for safety. Erectile dysfunction may be a manifestation of peripheral neuropathy, but the priority is still the client's safety.

3. When caring for a client receiving chemotherapy, the nurse plans care during the nadir of bone marrow activity to prevent which complication? A Drug toxicity B Polycythemia C Infection D Dose-limiting side effects

ANS: C The lowest point of bone marrow function is referred to as the nadir; risk for infection is highest during this phase. Drug toxicity can develop when drug levels exceed peak concentrations. Polycythemia refers to an increase in the number of red blood cells; typically chemotherapy causes reduction of red blood cells or anemia. Dose limiting side effects occur when the dose or frequency of chemotherapy need to be altered or held, such as in the case of severe neutropenia or neurologic dysfunction .

When caring for a client who has had a colostomy created during treatment for colon cancer, which nursing actions help support the client in accepting changes in appearance or function? (Select all that apply.) A Explain to the client that the colostomy is only temporary. B Encourage the client to participate in changing the ostomy C Obtain a psychiatric consultation. D Offer to have a person who is coping with a colostomy visit. E Encourage the client and family members to express their feelings and concerns.

ANS: B, D, E Encouraging the client to participate in changing the ostomy is an appropriate way for the client to become familiar with the ostomy and its care. A visit from a person who is successfully coping with an ostomy can demonstrate to the client that many aspects of life can be the same after surgery. Offering to listen to feelings and concerns is part of a therapeutic relationship and therapeutic communication. Ostomies may be temporary for bowel rest, such as after a perforation, but are typically permanent for cancer treatment. Obtaining a psychiatric consultation may need to be done for clients with persistent depression, but would not be done immediately.

Which finding alarms the nurse when caring for a client receiving chemotherapy who has a platelet count of 17,000/mm3? A Increasing shortness of breath B Diminished bilateral breath sounds C Change in mental status D Weight gain of 4 pounds in 1 day

ANS: C A change in mental status could result from spontaneous bleeding; in this case, a cerebral hemorrhage may have developed. Increasing shortness of breath is typically related to anemia, not to thrombocytopenia. Diminished breath sounds may be related to many factors, including poor respiratory excursion, infection, and atelectasis, which is not related to thrombocytopenia. A large weight gain in a short period may be related to kidney or heart failure; bleeding is the major complication of thrombocytopenia.

Which instruction is most appropriate for the nurse to convey to the client with chemotherapy-induced neuropathy? A Bathe in cold water. B Wear cotton gloves when cooking. C Consume a diet high in fiber. D Make sure shoes are snug.

ANS: C A high-fiber diet will assist with constipation due to neuropathy. The client should bathe in warm water, not hotter than 96° F. Cotton gloves may prevent harm from scratching; protective gloves should be worn for washing dishes and gardening. Wearing cotton gloves while cooking can increase the risk for burns. Shoes should allow sufficient length and width to prevent blisters. Shoes that are snug can increase the risk for blisters in a client with peripheral neuropathy.

A newly graduated RN has just finished a 6-week orientation to the oncology unit. Which client is most appropriate to assign to the new graduate? A A 30-year-old with acute lymphocytic leukemia who will receive combination chemotherapy today B A 40-year-old with chemotherapy-induced nausea and vomiting who has had no urine output for 16 hours C A 45-year-old with pancytopenia who will require IV administration of erythropoietin (Procrit) D A 72-year-old with tumor lysis syndrome who is receiving normal saline IV at a rate of 250 mL/hr

ANS: C A new nurse after a 6-week oncology orientation possesses the skills to care for clients with pancytopenia and with administration of medications to correct anemia. The clients with acute lymphocytic leukemia and chemotherapy-induced nausea are complex clients requiring a nurse certified in chemotherapy administration. The client with tumor lysis syndrome has complicated needs for assessment and care and should be cared for by an RN with more oncology experience.

2. Which statement made by a client allows the nurse to recognize whether the client receiving brachytherapy for ovarian cancer understands the treatment plan? A "I may lose my hair during this treatment." B "I must be positioned in the same way during each treatment." C "I will have a radioactive device in my body for a short time." D "I will be placed in a semiprivate room for company."

ANS: C Brachytherapy refers to short-term insertion of a radiation source. Side effects of radiation therapy are site-specific; this client is unlikely to experience hair loss from treating ovarian cancer with radiation. The client undergoing teletherapy (external beam radiation), not brachytherapy, must be positioned precisely in the same position each time. The client who is receiving brachytherapy must be in a private room.

The oncology nurse should use which intervention to prevent disseminated intravascular coagulation (DIC)? A Monitoring platelets B Administering packed red blood cells C Using strict aseptic technique to prevent infection D Administering low-dose heparin therapy for clients on bedrest

ANS: C Sepsis is a major cause of DIC, especially in the oncology client. Monitoring platelets will help detect DIC, but will not prevent it. Red blood cells are used for anemia, not for bleeding/coagulation disorders. Heparin may be administered to clients with DIC who have developed clotting, but this has not been proven to prevent the disorder.

The nurse corrects the nursing student when caring for a client with neutropenia secondary to chemotherapy in which circumstance? A The student scrubs the hub of IV tubing before administering an antibiotic. B The nurse overhears the student explaining to the client the importance of handwashing. C The student teaches the client that symptoms of neutropenia include fatigue and weakness. D The nurse observes the student providing oral hygiene and perineal care.

ANS: C Symptoms of neutropenia include low neutrophil count, fever, and signs and symptoms of infection; the student should be corrected. Asepsis with IV lines is an appropriate action. Handwashing is an essential component of client care, especially when the client is at risk for neutropenia. Hygiene and perineal care help prevent infection and sepsis.

When caring for the client with chemotherapy-induced mucositis, which intervention will be most helpful? A Administering a biological response modifier B Encouraging oral care with commercial mouthwash C Providing oral care with a disposable mouth swab D Maintaining NPO until the lesions have resolved

ANS: C The client with mucositis would benefit most from oral care; mouth swabs are soft and disposable and therefore clean and appropriate to provide oral care. Biological response modifiers are used to stimulate bone marrow production of immune system cells; mucositis or sores in the mouth will not respond to these medications. Commercial mouthwashes should be avoided because they may contain alcohol or other drying agents that may further irritate the mucosa. Keeping the client NPO is not necessary because nutrition is important during cancer treatment; a local anesthetic may be prescribed for comfort.

A client who is undergoing chemotherapy for breast cancer reports problems with concentration and memory. Which nursing intervention is indicated at this time? A Explain that this occurs in some clients and is usually permanent. B Inform the client that a small glass of wine may help her relax. C Protect the client from infection. D Allow the client an opportunity to express her feelings.

ANS: D Although no specific intervention for this side effect is known, therapeutic communication and listening may be helpful to the client. Evidence regarding problems with concentration and memory loss with chemotherapy is not complete, but the current thinking is that this process is usually temporary. The client should be advised to avoid the use of alcohol and recreational drugs at this time because they also impair memory. Chemotherapeutic agents are implicated in central nervous system function in this scenario, not infection.

4. The nurse teaches a client that intraperitoneal chemotherapy will be delivered to which part of the body? A Veins of the legs B Lung C Heart D Abdominal cavity

ANS: D Intraperitoneal chemotherapy is placed in the peritoneal cavity or the abdominal cavity. Intravenous drugs are delivered through veins. Chemotherapy delivered into the lungs is typically placed in the pleural space (intrapleural). Chemotherapy is not typically delivered into the heart.

An outpatient client is receiving photodynamic therapy. Which environmental factor is a priority for the client to adjust for protection? A Storing drugs in dark locations at room temperature B Wearing soft clothing C Wearing a hat and sunglasses when going outside D Reducing all direct and indirect sources of light

ANS: D Lighting of all types must be kept to a minimum with clients receiving photodynamic therapy; it can lead to burns of the skin and damage to the eyes because these clients' eyes are sensitive to light. Any drug that the client is prescribed should be considered for its photosensitivity properties; drugs should be stored according to the recommendations, but this is not the primary concern for this client. Clothing must cover the skin to prevent burns from direct or indirect light; texture is not a concern for the client receiving this treatment. The client will be homebound for 1 to 3 months after the treatment and should not go outside.

The nurse is teaching a client who is receiving an antiestrogen drug about the side effects she may encounter. Which side effects does the nurse include in the discussion? (Select all that apply.) A Heavy menses B Smooth facial skin C Hyperkalemia D Breast tenderness Correct E Weight loss F Deep vein thrombosis Correct

ANS: D, F Breast tenderness and shrinking breast tissue may occur with antiestrogen therapy. Venous thromboembolism may also occur. Irregular menses or no menstrual period is the typical side effect of antiestrogen therapy. Acne may also develop. Hypercalcemia, not hyperkalemia, is typical. Fluid retention with weight gain may also occur.

A client is preparing to give a client an antipyretic drug for a temperature of 101° F (38.3° C). What drug would be the most appropriate for the nurse to administer? Acetaminophen Aspirin Doxycycline Ibuprofen

Acetaminophen While all of these drugs may reduce fever, acetaminophen is an antipyretic and analgesic agent that has less side/adverse effects than the other drugs. Doxycycline is an antibiotic which may treat infection and thus reduce fever.

A client who is admitted with urolithiasis reports "spasms of intense flank pain, nausea, and severe dizziness." Which intervention does the nurse implement first? Administer morphine sulfate as prescribed. Infuse 0.9% normal saline at 100 mL/hr as prescribed Obtain a urine specimen for urinalysis as prescribed. Begin an infusion of metoclopramide as prescribed.

Administer morphine sulfate as prescribed.

Which assessment findings will the nurse expect to see in a client who is suspected to have systemic lupus erythematosus (SLE)? (Select all that apply.) Anemia Joint pain and swelling Hair loss Fever Fatigue Facial redness

Anemia Joint pain and swelling Hair loss Fever Fatigue Facial redness Each of these assessment findings has been associated with systemic lupus erythematosus (SLE).

Which type of drug therapy will the nurse prepare a client in the early disseminated stage of Lyme disease to take for control or cure of this disease? Convalescent serum Corticosteroids Biological response modifiers Antibiotics

Antibiotics The goal of therapy during the initial and disseminated stages of Lyme disease is to eradicate the organism causing the infection with antibiotic therapy. Common antibiotics prescribed, sometimes for up to 30 days, include doxycycline, amoxicillin, and erythromycin. None of the other types of therapy listed are focused on this outcome.

Which action is the priority for the nurse to take to prevent harm for the alert 58-year-old client who is admitted to the emergency department with wheezing, dyspnea, angioedema, blood pressure of 70/52 mm Hg, and an irregular apical pulse of 122 beats/min? Asking about exposure to possible allergens Applying oxygen via a high-flow nonrebreather mask at 90% to 100% Reassuring the client that appropriate interventions are being instituted Starting an IV infusion of normal saline

Applying oxygen via a high-flow nonrebreather mask at 90% to 100% The immediate priority is to apply oxygen in order to provide adequate oxygenation for the client who is in respiratory distress. Raising the lower extremities, starting an IV infusion, and reassuring the client are not the first priority for a client in respiratory distress.

What is the nurse's best response to a 38-year-old client with a large wound who does not want to receive a tetanus toxoid vaccination because he had a tetanus shot just 1 year ago? A)"Tetanus is a more serious disease and a "booster" is required every year to ensure adequate immunity and protection against it." B)"You may not need this vaccination now, I will check with your health care provider." C)"You need this vaccination because the strain of tetanus changes every year." D)"Because antibody production slows down as you age, it is better to take this vaccination as a booster to the one you had a year ago."

B) "You may not need this vaccination now, I will check with your health care provider." When people have been "boosting" their tetanus antibodies on a regularly scheduled basis, they should have sufficient circulating antibodies to mount a defense against exposure to tetanus. If this client's medical records substantiate that he did indeed receive a tetanus toxoid booster 1 year ago, he does not need another one now.

Which client health problems will the nurse identify as an infectious process along with inflammation rather than inflammation alone? (Select all that apply.) A) Tendonitis B) Appendicitis C) Asthma D) Cystitis E) Anaphylaxis F) Sepsis

B) Appendicitis D) Cystitis F) Sepsis Appendicitis is most commonly the result of an infectious process (usually bacterial), as is sepsis, although a widespread inflammatory response can accompany sepsis. Cystitis is a bladder infection most often with a bacterial infection cause. s are commonly caused by bacterial and viral infections. Tendonitis usually is a result of a closed or overuse injury and is characterized by inflammation without infection. Anaphylaxis is an allergic response, not an infection. Asthma is an irritant/allergic reaction, not an infection although a respiratory infection makes asthma worse.

Which assessment finding on a client with no other health problems does the nurse consider the greatest potential threat to the client's immune system? A) Has old scar formation related to an appendectomy. B) Has poor oral hygiene and numerous dental caries. C) Displays orthostatic hypotension and is mildly dehydrated. D) Displays occasional skipped heartbeats during auscultation.

B) Has poor oral hygiene and numerous dental caries. Poor oral hygiene and untreated dental carries are sources of infectious organisms with access to the blood (because mucous membranes are no longer intact). This is a potential threat to immunity because the condition can cause chronic inflammation and a constant transfer of microorganisms to the bloodstream, which increases the risk for systemic infection.

Which client laboratory response indicates to the nurse that granulocyte colony-stimulating factor therapy is successful? A) Increased lymphocytes B) Increased white blood cells C) Increased platelets D) Increased red blood cells

B) Increased white blood cells Granulocyte colony-stimulating factor is a growth factor that stimulates the increased production and maturation of neutrophils. This action increases the circulating number of neutrophils and has minimal effect on other blood cell types.

How will the nurse interpret a client's white blood cell count that has a total count of 9000 cells/mm3 (9 x 109/L) with a lymphocyte count of 4200 cells/mm3 (4.2 × 109/L)? A)The count indicates the client has an increased risk for infection. B) The client most likely has a viral infection. C)The count is completely normal. D)The client most likely has a bacterial infection.

B) The client most likely has a viral infection. Although the total white blood cell count is within the normal range, the lymphocyte count is elevated. The most common cause of lymphocyte count elevation is an actual viral infection. Bacterial infections are associated with higher total counts and higher neutrophil counts.

What is the most important precaution for the nurse to teach a client who has few natural killer cells and the natural killer cells are not very active? A)"You will need to avoid people with viral infections because it is harder now for you to develop antibodies." B)"You will need to have yearly checkups because your risk for cancer development is greater now." C)"You will be at an increased risk for developing allergies, so it will be necessary for you to avoid common allergens." D)"You will no longer develop a fever when you have an infection, so you must learn to identify other symptoms of infection."

B)"You will need to have yearly checkups because your risk for cancer development is greater now." Natural killer cells provide protection against development of cancer by recognizing unhealthy or cancer cells as non-self and taking action to destroy them.

Which type of immunity will the nurse initiate by administering an infusion of IV immunoglobulin to a client? A)Natural active immunity B)Artificial passive immunity C)Artificial active immunity D)Natural passive immunity

B)Artificial passive immunity The client will be receiving antibodies made in the body of another person and thus, is not actively involved in the production of these antibodies. That makes the immunity passive rather than active. Because the client is making the antibodies in response to an injection (vaccination) rather than in response to actually being sick with influenza, the immunity is artificial.

An 85-year-old client tells the nurse that she does not perform breast self-exam because there is no history of breast cancer in her family. What is the nurse's best response? A. "Because your breasts are no longer as dense as they were when you were younger, your risk for breast cancer is now decreased." B. "Breast cancer can be found more frequently in families; however, the risk for general, nonfamilial breast cancer increases with age." C. "You are correct. Breast cancer is an inherited type of malignancy and your family history indicates a low risk for you." D. "Examining your breasts once per year when you have your mammogram is sufficient screening for someone with your history."

B. "Breast cancer can be found more frequently in families; however, the risk for general, nonfamilial breast cancer increases with age." Rationale:The risks for all types of sporadic (noninherited, nonfamilial) cancers increase with age. An 85-year-old woman is two to three times more likely to have breast cancer than is a 30-year-old woman.

The nurse wishes to reduce the incidence of hospital-acquired acute kidney injury. Which question by the nurse to the interdisciplinary health care team will result in reducing client exposure? A. "Should we filter air circulation?" B. "Can we use less radiographic contrast dye?" C. "Should we add low-dose dobutamine?" D. "Should we decrease IV rates?"

B. "Can we use less radiographic contrast dye?"

The nurse discusses the importance of restricting sodium in the diet for a client with heart failure. Which client statement indicates the need for further teaching? A. "I won't put the salt shaker on the table anymore." B. "I need to avoid eating hamburgers." C. "I need to avoid lunchmeats but may cook my own turkey." D. "I must cut out bacon and canned foods."

B. "I need to avoid eating hamburgers." Cutting out beef or hamburgers made at home is not necessary; however, fast-food hamburgers are to be avoided owing to higher sodium content. Bacon, canned foods, lunchmeats, and processed foods are high in sodium, which promotes fluid retention; these are to be avoided.

The nurse is caring for a client with heart failure who is prescribed spironolactone. Which client statement requires further nursing education? A. "I may need to take this drug every other day according to lab values." B. "I need to take potassium supplements with this medication." C. "I will try my best not to use table salt on my food." D. "This medication will cause me to urinate more often."

B. "I need to take potassium supplements with this medication." Rationale: Spironolactone is a potassium sparing diuretic. This drug can cause hyperkalemia and as such the client would not take potassium supplements with this drug. The statement, "I need to take potassium supplements with this medication" requires additional nursing education. It is appropriate to take the medication daily at the same time, to avoid table salt on food, and the medication does cause increased urination as it is a diuretic.

The nurse is teaching a client's family regarding the diagnosis of septic shock. Which teaching will the nurse include? Select all that apply. A. "The blood cultures will tell us for sure if your loved one has septic shock." B. "The client's change in behavior and lethargy may be associated with septic shock." C. "Antibiotics, as prescribed, will be started within the hour to treat the sepsis." D. "An insulin drip has been started to keep the client's glucose as low as possible." E. "Septic shock is easily treated with multiple antibiotics."

B. "The client's change in behavior and lethargy may be associated with septic shock." C. "Antibiotics, as prescribed, will be started within the hour to treat the sepsis."

A client awaiting kidney transplantation states, "I can't stand this waiting for a kidney, I just want to give up." Which statement by the nurse is most therapeutic? A. "I'll talk to the health care provider and have your name removed from the waiting list." B. "You sound frustrated with the situation." C. "You're right, the wait is endless for some people." D. "I'm sure you'll get a phone call soon that a kidney is available."

B. "You sound frustrated with the situation."

A 40-year-old man who has a mother who was diagnosed with breast cancer at age 45, a father who was diagnosed with smoking-related lung cancer at age 55, a 33-year-old sister with breast cancer, and a 38-year-old sister with ovarian cancer, asks if he should be concerned for his cancer risk. What is the nurse's best response? A. "You have two first-degree relatives and two second-degree relatives with cancer, which increases your general risk for cancer." B. "Your risk for breast cancer is increased; however, your risk for lung cancer is not affected by this history." C. "Your risk for cancer is affected by your parents' cancer development; your sisters' cancers have no bearing on your risk." D. "Your risk is not affected by this family history because most of the cancers arose in female sex-associated tissues."

B. "Your risk for breast cancer is increased; however, your risk for lung cancer is not affected by this history." Rationale:This man has four first-degree relatives with cancer, three of whom have cancers that are associated with a genetic risk. The fact that the sisters and mother were diagnosed at relatively young ages increases the likelihood of a genetic predisposition. The genetic association with these cancers also increases the risk for male members of the family. Lung cancer has not been found to have a genetic association.

Based on the assessment data, which client will the nurse identify as having a higher risk for development of sepsis and septic shock? (Select all that apply.) A. A 41-year-old male client with a closed fracture of the femur. B. A 54-year-old female with breast cancer who is receiving chemotherapy. C. An 86-year-old male with acute onset confusion. D. A 44-year-old male client who has a history of alcoholism and diabetes mellitus. E. A 44-year-old female client with a history of anxiety and infertility. F. A 38-year-old male with HIV who has a low viral load. G. A 40-year-old female with a history of a double lung transplant 4 years ago.

B. A 54-year-old female with breast cancer who is receiving chemotherapy. C. An 86-year-old male with acute onset confusion. D. A 44-year-old male client who has a history of alcoholism and diabetes mellitus. F. A 38-year-old male with HIV who has a low viral load. G. A 40-year-old female with a history of a double lung transplant 4 years ago.

A 74-year-old client recovering from lung cancer surgery tells the nurse, "I don't understand why I have lung cancer. I have never even touched a cigarette." Which factor may explain the cause? A. A history of cardiac disease B. Advancing age C. A history of military service D. A diagnosis of diabetes

B. Advancing age Rationale:Advancing age is the single most important risk factor for cancer. As a person ages, immune protection decreases and therefore risk for overgrowth of cancer cells increases. Diabetes is not known to cause lung cancer. A history of cardiac disease does not predispose a person to lung cancer, nor does a history of military service.

The nurse is assessing a client with right-sided heart failure. What assessment findings will the nurse anticipate? (Select all that apply.) A. Oliguria B. Ascites C. Pulmonary congestion D. Peripheral edema E. Shortness of breath F. Third heart sound

B. Ascites D. Peripheral edema Rationale: - Clients diagnosed with right-sided heart failure generally retain fluid in interstitial body tissues and the abdomen due to portal (liver) congestion. Assessment findings related to this fluid gain include increased body weight, peripheral edema, and ascites (the accumulation of fluid in the peritoneal cavity). Lung congestion causing crackles and shortness of breath or breathlessness results from left-sided heart failure because fluid backs up from the left heart into the lungs.

Which instruction by the nurse will help a client with chronic kidney disease prevent renal osteodystrophy? A. Consuming a low-calcium diet B. Avoiding peas, nuts, and legumes C. Drinking cola beverages only once daily D. Increasing dairy products enriched with vitamin D

B. Avoiding peas, nuts, and legumes

The nurse recognizes that a client's hemangiosarcoma originated in which tissue? A. Epithelial tissue B. Blood vessel C. Skeletal muscle D. Cartilage

B. Blood vessel Rationale:The prefix "hemangio-" is included when cancers of the blood vessel are named.The prefix "rhabdo-" is used when cancers of the skeletal muscle are named.The prefix "chondro-" is included when cancers of cartilage are named. The prefix "adeno-" is included when cancers of epithelial tissues are named

A client has just returned from coronary artery bypass graft surgery. Which assessment data requires immediate nursing action? A. Temperature 98.2° F (36.8° C) B. Chest tube drainage 175 mL last hour C. Serum potassium 3.9 mEq/L (3.9 mmol/L) D. Incisional pain 6 on a scale of 0-10

B. Chest tube drainage 175 mL last hour

Which client demonstrates the highest risk for hypovolemic shock? A. Client receiving a blood transfusion B. Client with severe ascites C. Client with myocardial infarction D. Client with syndrome of inappropriate antidiuretic hormone (SIADH) secretion

B. Client with severe ascites

The nurse in the coronary care unit is caring for a group of clients who have had a myocardial infarction. Which client will the nurse see first? A. Client with normal sinus rhythm and PR interval of 0.28 second B. Client with third-degree heart block on the monitor C. Client with dyspnea on exertion when ambulating to the bathroom D. Client who refuses to take heparin or nitroglycerin

B. Client with third-degree heart block on the monitor

Which assessment data cause the nurse to suspect that a client who had a myocardial infarction (MI) is developing cardiogenic shock? (Select all that apply.) A. Bradycardia B. Cool, diaphoretic skin C. Crackles in the lung fields D. Respiratory rate of 12 breaths/min E. Anxiety and restlessness F. Temperature of 100.4° F (38.0° C)

B. Cool, diaphoretic skin C. Crackles in the lung fields E. Anxiety and restlessness

A client is diagnosed with left-sided heart failure. Which assessment findings will the nurse expect the client to have? Select all that apply. A. Peripheral edema B. Crackles in both lungs C. Tachycardia D. Ascites E. Tachypnea F. S3 gallop

B. Crackles in both lungs C. Tachycardia E. Tachypnea F. S3 gallop Rationale: For a client with left sided heart failure the nurse will anticipate assessment findings of crackles in both lungs, tachypnea, tachycardia, and a third heart sound, usually an S3 gallop. Peripheral edema and ascites are associated with right sided heart failure.

The client in the cardiac care unit has had a large myocardial infarction. What assessment data indicates to the nurse the onset of left ventricular failure? A. Urine output of 1500 mL on the preceding day B. Crackles in the lung fields C. Pedal edema D. Expectoration of yellow sputum

B. Crackles in the lung fields

Which atypical symptoms may be present in a female client experiencing myocardial infarction (MI)? (Select all that apply.) A. Sharp, inspiratory chest pain B. Dyspnea C. Dizziness D. Extreme fatigue E. Anorexia

B. Dyspnea C. Dizziness D. Extreme fatigue

The nurse plans to administer an antibiotic to a client newly admitted with septic shock. What action will the nurse take first? A. Administer the antibiotic. B. Ensure that blood cultures were drawn. C. Insert an intravenous line. D. Take the client's vital signs.

B. Ensure that blood cultures were drawn.

A client is receiving immune-suppressive therapy after kidney transplantation. Which measure for infection control is most important for the nurse to implement? A. Adherence to therapy B. Handwashing C. Monitoring for low-grade fever D. Strict clean technique

B. Handwashing

Which actions or behaviors represent to the nurse that a client is engaging in secondary cancer prevention practices? (Select all that apply.) A. Eating a diet high in fiber and low in animal fat B. Having a health checkup, including chest x-ray, annually C. Obtaining a colonoscopy every 5 years D. Electing to have both ovaries removed a person who has a BRCA2 mutation E. Getting a mammogram or breast MRI annually F. Having a mole removed from the neck

B. Having a health checkup, including chest x-ray, annually E. Getting a mammogram or breast MRI annually

The nurse is preparing to administer digoxin as prescribed to a client with heart failure and notes: Temperature: 99.8° F (37.7° C), Pulse: 48 beats/min and irregular, Respirations: 20 breaths/min, Potassium level: 3.2 mEq/L (3.2 mmol/L). What action will the nurse take? A. Hold the digoxin, and obtain a prescription for an additional dose of furosemide. B. Hold the digoxin, and obtain a prescription for a potassium supplement. C. Give the digoxin; document assessment findings in the medical record. D. Give the digoxin; reassess the heart rate in 30 minutes.

B. Hold the digoxin, and obtain a prescription for a potassium supplement. Rationale: Digoxin causes bradycardia; hypokalemia potentiates digoxin. Because digoxin causes bradycardia, the medication should be held. Furosemide decreases circulating blood volume and depletes potassium.

Which characteristics place women at high risk for myocardial infarction (MI)? (Select all that apply.) A. Premenopausal B. Increasing age C. Family history D. Abdominal obesity E. Breast cancer

B. Increasing age C. Family history D. Abdominal obesity

How does the nurse caring for a client with septic shock recognize that severe tissue hypoxia is present? A. PaCO2 58 mm Hg B. Lactate 81 mg/dL (9.0 mmol/L) C. Partial thromboplastin time 64 seconds D. Potassium 2.8 mEq/L (2.8 mmol/L)

B. Lactate 81 mg/dL (9.0 mmol/L)

Which medication is most effective in slowing the progression of kidney failure in a client with chronic kidney disease? A. Diltiazem (Cardizem) B. Lisinopril (Zestril) C. Clonidine (Catapres) D. Doxazosin (Cardura)

B. Lisinopril (Zestril)

Which clinical symptoms in a postoperative client indicate early sepsis with an excellent recovery rate if treated? A. Localized erythema and edema B. Low-grade fever and mild hypotension C. Low oxygen saturation rate and decreased cognition D. Reduced urinary output and increased respiratory rate

B. Low-grade fever and mild hypotension

A postoperative client is admitted to the intensive care unit (ICU) with hypovolemic shock. Which nursing action will the nurse delegate to an experienced assistive personnel (AP)? A. Obtain vital signs every 15 minutes. B. Measure hourly urine output. C. Check oxygen saturation. D. Assess level of alertness.

B. Measure hourly urine output.

A client comes to the emergency department with chest discomfort. Which action does the nurse perform first? A. Administers oxygen therapy. B. Obtains the client's description of the chest discomfort. C. Provides pain relief medication. D. Remains calm and stays with the client.

B. Obtains the client's description of the chest discomfort.

The nurse is caring for a hospitalized client with infective endocarditis who has been receiving antibiotics for 2 days. The client is now experiencing flank pain with hematuria. What condition will the nursing suspect? A. Pulmonary embolus B. Renal infarction C. Transient ischemic attack D. Splenic infarction

B. Renal infarction Rationale: The classic clinical signs of renal infarction, associated with embolization from infective endocarditis, are flank pain, hematuria, and pyuria.

When caring for a client with acute kidney injury and a temporary subclavian hemodialysis catheter, which assessment finding does the nurse report to the provider? A. Mild discomfort at the insertion site B. Temperature 100.8° F C. 1+ ankle edema D. Anorexia

B. Temperature 100.8° F

How will the nurse interpret the finding on a client's pathology report that a cancerous tumor has a mitotic index of 8%? A. The tumor has not yet undergone carcinogenesis. B. The tumor is slow-growing. C. Metastasis has already occurred. D. The tumor has an abnormal number of chromosomes.

B. The tumor is slow-growing. Rationale:A mitotic index of 8% means that only 8% of the cells within the tumor sample are actively dividing, which represents a low or slow growth rate. The presence or absence of metastasis cannot be determined by the mitotic index. By definition, a cancerous tumor has already undergone carcinogenesis, which is not determined by the mitotic index. When a tumor has an abnormal number of chromosomes, it is aneuploid, which is not related to the mitotic index.

A 45 year old male client having an annual physical asks the nurse about his risk for developing a myocardial infarction (MI). Which modifiable risk factors will the nurse assess to guide the client's teaching plan? (Select all that apply.) A. Age B. Tobacco use C. Gender D. Diet E. Family history F. Weight

B. Tobacco use D. Diet F. Weight

When caring for a client with a left forearm arteriovenous fistula created for hemodialysis, which actions must the nurse take? (Select all that apply.) A. Check brachial pulses daily. B. Auscultate for a bruit every 8 hours. C. Teach the client to palpate for a thrill over the site. D. Elevate the arm above heart level. E. Ensure that no blood pressures are taken in that arm.

BCE

Which signs and symptoms indicate rejection of a transplanted kidney? (Select all that apply.) Blood urea nitrogen (BUN) 21 mg/dL, creatinine 0.9 mg/dL Crackles in the lung fields Temperature of 98.8° F (37.1° C) Blood pressure of 164/98 mm Hg 3+ edema of the lower extremities

BDE

How do immune system cells differentiate between normal, healthy body cells and non-self cells within the body? A) All normal, healthy body cells are considered a part of the immune system. B) Immune system cells recognize normal healthy body cells by the presence of the nucleus, a structure that is lacking in non-self cells. C) Non-self cells express surface proteins that are different from normal, healthy body cells and are recognized as "foreign" by immune system cells. D) Non-self cells are easily identified by the immune system cells because non-self cells are much larger than normal, healthy body cells.

C) Non-self cells express surface proteins that are different from normal, healthy body cells and are recognized as "foreign" by immune system cells. Normal, healthy body cells all express surface proteins that are unique to the person, coded by the major histocompatibility genes. Non-self cells express different cell surface proteins. Immune system cells can distinguish between their own surface proteins and all others.

Which number will the nurse calculate as the absolute neutrophil count (ANC) for a client whose differential includes: total WBCs 5300/mm3 (5.3 × 109/L ); segs 2800/mm3 (2.8 × 109/L); bands 200/mm3 (0.20 × 109/L); monos 250/mm3 (0.25 × 109/L); lymphs 2000/mm3 (2.0 × 109/L); eosins 25/mm3 (0.025 × 109/L); basos 25 (0.025 × 109/L)? A)2800/mm3 (2.8 × 109/L) B)3200/mm3 (3.2 × 109/L) C)3000/mm3 (3.0 × 109/L) D)2300/mm3 (2.3 × 109/L)

C)3000/mm3 (3.0 × 109/L) The absolute neutrophil count is calculated by adding the mature neutrophil count (segs) with the slightly less mature band neutrophil count (which will mature within a matter of hours into segs). Monos, lymphs, eosins, and basos are not neutrophils.

A client who recently had a heart valve replacement is preparing for discharge. What statement by the client indicates that the nurse will need to do additional health teaching? A. "I need to brush my teeth at least twice daily and rinse with water." B. "I will eat foods that are low in vitamin K, such as potatoes and iceberg lettuce." C. "I need to take a full course of antibiotics prior to my colonoscopy." D. "I will take my blood pressure every day and call if it is too high or low."

C. "I need to take a full course of antibiotics prior to my colonoscopy." Rationale: Antibiotics are only required prior to dental procedures. Good oral hygiene is the best prevention for endocarditis. The statement in option A is correct and shows the patient understands the need for oral hygiene. The patient with a mechanical valve will be on warfarin thus, foods high in Vitamin K should be avoided. This statement in option B is correct and shows the patient understands foods that are LOW in Vitamin K. This statement in option D is also correct and shows that the patient understands the importance of regular BP assessment as well as when to call the provider based on the assessment.

A client is scheduled to undergo kidney transplant surgery. Which teaching point does the nurse include in the preoperative teaching session? A. "Your diseased kidneys will be removed at the same time the transplant is performed." B. "The new kidney will be placed directly below one of your old kidneys." C. "It is essential for you to wash your hands and avoid people who are ill." D. "You will receive dialysis the day before surgery and for about a week after."

C. "It is essential for you to wash your hands and avoid people who are ill."

Discharge teaching has been provided for a client recovering from kidney transplantation. Which information indicates that the client understands the instructions? A. "I can stop my medications when my kidney function returns to normal." B. "If my urine output is decreased, I should increase my fluids." C. "The anti-rejection medications will be taken for life." D. "I will drink 8 ounces of water with my medications."

C. "The anti-rejection medications will be taken for life."

A client undergoing coronary artery bypass grafting asks why the surgeon has chosen to use the internal mammary artery for the surgery. Which nursing response is appropriate? A. "This way you will not need to have a leg incision." B. "The surgeon prefers this approach because it is easier." C. "These arteries remain open longer." D. "The surgeon has chosen this approach because of your age."

C. "These arteries remain open longer."

After receiving change-of-shift report about these four clients, which client would the nurse assess first? A. A 79 year old admitted for possible rejection of a heart transplant who has sinus tachycardia, heart rate 104 beats/min. B. A 55 year old admitted with pulmonary edema who received furosemideand whose current O2 saturation is 94%. C. A 46 year old with aortic stenosis who takes digoxin and has new-onset frequent premature ventricular contractions. D. A 68 year old with pericarditis who is reporting sharp chest pain with inspiration.

C. A 46-year-old with aortic stenosis who takes digoxin and has new-onset frequent premature ventricular contractions. Rationale:The 46-year-old's premature ventricular contractions may be indicative of digoxin toxicity; further assessment for clinical manifestations of digoxin toxicity should be done and the health care provider notified about the dysrhythmia. The 55-year-old is stable and can be assessed after the client with aortic stenosis. The 68-year-old may be assessed after the client with aortic stenosis; this type of pain is expected in pericarditis. Tachycardia is expected in the 79-year-old because rejection will cause signs of decreased cardiac output, including tachycardia; this client may be seen after the client with aortic stenosis.

An LPN/LVN is scheduled to work on the stepdown cardiac unit. Which client will the charge nurse assign to the LPN/LVN? A. A 60 year old who was admitted today for pacemaker insertion because of third-degree heart block and who is now reporting chest pain. B. A 62 year old who underwent open-heart surgery 4 days ago for mitral valve replacement and who has a temperature of 100.8° F (38.2° C). C. A 66 year old who has a prescription for a nitroglycerin patch and is scheduled for discharge to a long-term care later today. D. A 69 year old who had a stent placed 2 hours ago in the left anterior descending artery and who has bursts of ventricular tachycardia.

C. A 66 year old who has a prescription for a nitroglycerin patch and is scheduled for discharge to a long-term care later today.

Which client is best to assign to an LPN/LVN working on the telemetry unit? A. Client with pericarditis who has a paradoxical pulse and distended jugular veins. B. Client with heart failure who is receiving dobutamine. C. Client with dilated cardiomyopathy who uses oxygen for exertional dyspnea. D. Client with rheumatic fever who has a new systolic murmur.

C. Client with dilated cardiomyopathy who uses oxygen for exertional dyspnea Rationale: The client with dilated cardiomyopathy who needs oxygen only with exertion is the most stable; administration of oxygen to a stable client is within the scope of LPN/LVN practice. The client with heart failure is receiving an intravenous inotropic agent, which requires monitoring by the RN. The client with pericarditis is displaying signs of cardiac tamponade and requires immediate lifesaving intervention. The client with a new-onset murmur requires assessment and notification of the provider, which is within the scope of practice of the RN.

The nurse is caring for a client in the refractory stage shock. Which intervention does the nurse consider? A. Admission to rehabilitation hospital for ambulatory retraining B. Collaboration with home care agency for return to home C. Discussion with family and provider regarding palliative care D. Enrollment in a cardiac transplantation program

C. Discussion with family and provider regarding palliative care

The nurse is teaching a group of teens about prevention of heart disease. Which point is most important for the nurse to emphasize? A. Reduce abdominal fat. B. Implement stress-reduction techniques. C. Do not smoke or chew tobacco. D. Avoid alcoholic beverages.

C. Do not smoke or chew tobacco.

A client with end-stage kidney disease has been put on fluid restrictions. Which assessment finding indicates that the client has not adhered to this restriction? A. Blood pressure of 118/78 mm Hg B. Weight loss of 3 pounds during hospitalization C. Dyspnea and anxiety at rest D. Central venous pressure (CVP) of 6 mm Hg

C. Dyspnea and anxiety at rest

When educating a client with B-cell lymphoma, a nurse tells the client that a virus can contribute to the development of their cancer. Which virus is linked with B-cell lymphoma? A. Human lymphotrophic virus type II B. Human papilloma virus C. Epstein-Barr virus D. Hepatitis B virus

C. Epstein-Barr virus Rationale:The Epstein-Barr virus has been associated with B-cell lymphoma, Burkitt lymphoma, and nasopharyngeal carcinoma. Hepatitis B, human papilloma virus, and human lymphotrophic virus type II are associated with other cancers, but are not associated with B-cell lymphoma

A client is diagnosed with melanoma. Which areas would the nurse anticipate that this client's tumor might metastasize? (Select all that apply.) A. Kidneys B. Liver C. Gastrointestinal tract D. Lymph nodes E. Brain F. Lungs

C. Gastrointestinal tract D. Lymph nodes E. Brain F. Lungs Rationale:Typical sites of metastasis for melanoma include brain, lymph nodes, lungs, and the gastrointestinal tract. Liver and kidneys are not typical sites for melanoma metastasis.

The nurse is reviewing the laboratory profile of a client with hypovolemic shock. What lab values will the nurse anticipate? A. pH 7.51 B. PaO2 106 mmHg C. PaCO2 49 mmHg D. Lactate 0.4 mmol/L

C. PaCO2 49 mmHg

1. The nurse is caring for a client with heart failure who is on oxygen at 2L per nasal cannula with an oxygen saturation of 90%. The client states, "I feel short of breath." Which action will the nurse take first? A. Contact respiratory therapy. B. Increase the oxygen to 4L. C. Place the client in a high Fowler's position. D.Draw arterial blood for arterial blood gas analysis.

C. Place the client in a high Fowler's position. Rationale: The first action of the nurse is to place the client in high Fowler's position. This position allows for maximal lung expansion. The nurse can also place pillows under each arm to maximize chest expansion. Repositioning the client with heart failure can improve overall gas exchange. If dyspnea continues the nurse may contact respiratory therapy for a breathing treatment, assess arterial blood gases (as prescribed) or increase oxygen if warranted by ABG results.

The nurse is caring for a client in phase 1 cardiac rehabilitation. Which activity does the nurse suggest? A. The need to increase activities slowly at home B. Planning and participating in a walking program C. Placing a chair in the shower for independent hygiene D. Consultation with social worker for disability planning

C. Placing a chair in the shower for independent hygiene

The nurse is caring for a client with heart failure in the coronary care unit. The client is exhibiting signs of air hunger and anxiety. Which nursing intervention will the nurse perform first for this client? A. Monitor and document heart rate, rhythm, and pulses. B. Encourage alternate rest and activity periods. C. Position the client to alleviate dyspnea. D. Determine the client's physical limitations.

C. Positions the client to alleviate dyspnea Rationale: Positioning the client to alleviate dyspnea will help ease air hunger and anxiety. Administering oxygen therapy is also an important priority action. Determining the client's physical limitations is not a priority in this situation.

Which problem places a client at highest risk for sepsis? A. Pernicious anemia B. Pericarditis C. Post kidney transplant D. Client owns an iguana

C. Post kidney transplant

A client admitted to the medical unit with a history of vomiting and diarrhea and an increased blood urea nitrogen requires 1 liter of normal saline infused over 2 hours. Which staff member should be assigned to care for the client? A. RN who has floated from pediatrics for this shift B. LPN/LVN with experience working on the medical unit C. RN who usually works on the general surgical unit D. New graduate RN who just finished a 6-week orientation

C. RN who usually works on the general surgical unit

The nurse is assessing a client with a cardiac infection. Which nursing assessment data causes the nurse to suspect infective endocarditis instead of pericarditis or rheumatic carditis? A. Thickening of the endocardium B. Pain aggravated by breathing, coughing, and swallowing C. Splinter hemorrhages D. Friction rub auscultated at the left lower sternal border

C. Splinter hemorrhages Rationale: Splinter hemorrhages are indicative of infective endocarditis. Friction rub in the left lower sternal border and pain aggravated by breathing, coughing, and swallowing are signs and symptoms indicative of chronic constrictive pericarditis. Thickening of the endocardium is indicative of rheumatic carditis.

Which nursing intervention for a client admitted today with heart failure will assist the client to conserve energy? A. The nurse monitors the client's pulse and blood pressure frequently. B. The client ambulates around the nursing unit with a walker. C. The nurse obtains a bedside commode before administering furosemide. D. The nurse returns the client to bed when the client becomes tachycardia.

C. The nurse obtains a bedside commode before administering furosemide.

While assisting a client during peritoneal dialysis, the nurse observes the drainage stop after 200 mL of peritoneal effluent drains into the bag. What action should the nurse implement first? A. Instruct the client to deep-breathe and cough. B. Document the effluent as output. C. Turn the client to the opposite side. D. Re-position the catheter.

C. Turn the client to the opposite side.

Which factor represents a sign or symptom of digoxin toxicity? A. Serum digoxin level of 1.2 ng/mL B. Polyphagia C. Visual changes D. Serum potassium of 5.0 mEq/L

C. Visual changes

What is the pathophysiologic basis for Lyme disease progression to stage III? Changing the organism's surface antigens leading to chronic inflammation and elevated cytokine levels Failure of the immune system to recognize the causative organism as non-self, allowing it to become a systemic infection Triggering of antibodies against infected cells that lead to autoimmune disease The special ability of Borrelia burgdorferi to burrow deeply into joint, cardiac, and neurons causing direct damage to these tissues.

Changing the organism's surface antigens leading to chronic inflammation and elevated cytokine levels The causative organisms can switch out parts of its unique surface proteins, which changes the ability of immune sensitized system cells and antibodies to recognize the existing infecting organism allowing it to "hide." Every time a switch occurs, the immune system treats them like a new infection, and develops new antibodies and inflammatory responses to them, resulting in keeping all general and specific immunity actions in continual but ineffective attack mode through all stages of the disease process. This prolonged and continuous process results in persistent and enhanced damage to a variety of tissues and organs.

A 73-year-old woman is admitted to the telemetry unit with Class III left-sided heart failure, type 2 diabetes mellitus and hypertension. When the patient arrives to the unit, the nurse assesses that her color is pale, she is becoming increasingly dyspneic, and appears restless. The patient's husband is very concerned because he thinks his wife is slightly disoriented. The patient has crackles in bilateral lung bases and her oxygen saturation levels are 88% on oxygen at 2 liters per nasal cannula. What client assessment would indicate that the nurse's actions were effective? (Hint: Think about signs that would indicate an improvement, decline, or unchanged client condition.)

Client assessment data that indicates improvement include: decreasing dyspnea, oxygen saturations greater than 90%, increasing urine output from diuretic administration. Assessment data that indicates a decline would include increasing dyspnea even with interventions, a cough with frothy or blood-tinged sputum, and increasing air hunger and anxiety.

During discharge planning after admission for a myocardial infarction, the client says, "I won't be able to increase my activity level. I live in an apartment, and there is no place to walk." Which nursing response is appropriate? A. "You are right. Focus more on your diet." B. "You must find someplace to walk." C. "Walk around the edge of your apartment complex." D. "Where might you be able to walk?"

D. "Where might you be able to walk?"

The nurse is caring for a client with hypovolemic shock that is bleeding from a traumatic injury to the upper chest wall. What is the priority nursing action? A. Insert a large bore IV catheter. B. Administer supplemental oxygen. C. Elevate the client's feet, keeping the head flat. D. Apply direct pressure to the area of overt bleeding.

D. Apply direct pressure to the area of overt bleeding.

When caring for a client who is obtunded and admitted with shock of unknown origin, which action will the nurse take first? A. Obtain IV access and hang prescribed fluid infusions. B. Apply the automatic blood pressure cuff. C. Assess level of consciousness and pupil reaction to light. D. Check the airway and respiratory status.

D. Check the airway and respiratory status.

The nurse is caring for a postoperative client at risk for hypovolemic shock. Which assessment indicates an early sign of shock? A. Blood pressure 100/48 mm Hg B. Respiratory rate 12 breaths/min C. First-degree heart block D. Heart rate 120 beats/min

D. Heart rate 120 beats/min

Which assessment finding represents a positive response to erythropoietin (Epogen, Procrit) therapy? A. Hematocrit of 26.7% B. Potassium within normal range C. Absence of spontaneous fractures D. Less fatigue

D. Less fatigue

The nurse is providing discharge teaching to a client with heart failure, focusing on when to seek medical attention. Which client statement indicates understanding of the teaching? A. "I should expect occasional chest pain." B. "I will try walking for 1 hour each day." C. "I will report to the provider weight loss of 2 to 3 lb (0.9 to 1.4 kg) in a day." D. "I will call the provider if I have a cough lasting 3 or more days."

D. "I will call the provider if I have a cough lasting 3 or more days." Rationale: Cough, a symptom of heart failure, is indicative of intra-alveolar edema; the provider should be notified. The client should call the provider for weight gain of 3 pounds in a week.

The nurse is caring for a client who is scheduled for a percutaneous transluminal angioplasty (PTCA). Which client statement indicates a need for further teaching? A. "I will be awake during this procedure." B. "I will have a balloon in my artery to widen it." C. "I must lie still after the procedure." D. "My angina will be gone for good."

D. "My angina will be gone for good."

The nurse is caring a college athlete who collapsed during soccer practice. The client has been diagnosed with hypertrophic cardiomyopathy and states, "This can't be. I am in great shape. I eat right and exercise." Which nursing response is appropriate? A. "How does this make you feel?" B. "This can be caused by taking performance-enhancing drugs." C. "It could be worse if you weren't in good shape." D. "This may be caused by a genetic trait."

D. "This may be caused by a genetic trait." Rationale: Hypertrophic cardiomyopathy is often transmitted as a single gene autosomal dominant trait. Exploring the client's feelings is important, but does not address the client's question. Hypertrophic cardiomyopathy is not caused by performance-enhancing drugs. Reminding the client that he or she is in good shape is not at all therapeutic and does not address the client's question.

Which problem excludes a client hoping to receive a kidney transplant from undergoing the procedure? A. History of hiatal hernia B. Presence of diabetes and glycosylated hemoglobin of 6.8% C. History of basal cell carcinoma on the nose 5 years ago D. Presence of tuberculosis

D. Presence of tuberculosis

A client is being treated for kidney failure. Which statement by the nurse encourages the client to express his or her feelings and concerns about the risk for death and the disruption of lifestyle? A. "All of this is new. What can't you do?" B. "Are you afraid of dying?" C. "How are you doing this morning?" D. "What concerns do you have about your kidney disease?"

D. "What concerns do you have about your kidney disease?"

The nurse is caring for a client 36 hours after coronary artery bypass grafting. Which assessment causes the nurse to terminate an activity and return the client to bed? A. HR 72 beats/min and regular B. Urinary frequency C. Incisional discomfort D. Respiratory rate 28 breaths/min

D. Respiratory rate 28 breaths/min

Which new assessment finding in a client being treated for hypovolemic shock indicates to the nurse that interventions are currently effective? A. Oxygen saturation remains unchanged. B. Core body temperature has increased to 99° F (37.2° C). C. The client correctly states the month and year. D. Serum lactate and serum potassium levels are declining.

D. Serum lactate and serum potassium levels are declining.

Which cell types provide protective responses during inflammation? A) Natural killer cell B) Basophils C) Eosinophils D) Platelets E) Macrophages

D) Platelets Macrophages and neutrophils initiate and complete phagocytosis against invading microorganism, providing the body with protection against infection. Natural killer cells are not particularly active during inflammation. Eosinophils and basophils are responsible for vascular changes, not protection. Platelets have no direct role in the protection provided by inflammation.

Which differential count will the nurse report to the primary health care provider for a client whose white blood count indicates a total count of 10,000 cells/mm3 (10 × 109/L)? A)Eosinophils 200/mm3 (0.2 × 109/L) B)Lymphocytes 2100/mm3 (2.1 × 109/L) C)Segmented neutrophils 6000/mm3 (6 × 109/L) D)Monocytes 2000/mm3 (2 × 109/L)

D)Monocytes 2000/mm3 (2 × 109/L) The normal monocyte population in peripheral blood should be not greater than 5%. A monocyte count of 2000 in 10,000 white blood cells represents 20% of the total and indicates a significant increase. Question 9 of 16

A client with heart failure is prescribed furosemide. Which assessment data concerns the nurse with this new prescription? A. Serum sodium level of 135 mEq/L (135 mmol/L) B. Serum magnesium level of 1.9 mEq/L (0.95 mmol/L) C. Serum creatinine of 1.0 mg/dL (88.4 mcmol/L) D. Serum potassium level of 2.8 mEq/L (2.8 mmol/L)

D. Serum potassium level of 2.8 mEq/L (2.8 mmol/L) Rationale: Clients taking loop diuretics should be monitored for potassium deficiency from diuretic therapy.A serum sodium level of 135 mEq/L is a normal value. serum creatinine of 1.0 mg/dL represents a normal value serum magnesium level of 1.9 mEq/L is a normal value.

Which client circumstance would prompt the nurse to create a three-generation pedigree to more fully explore the possibility of increased genetic risk for cancer? A. Smoked for 20 years but quit 5 years ago B. Personal history of excessive sun exposure C. Most family adult members are overweight D. Strong family history of breast cancer

D. Strong family history of breast cancer Rationale:Breast cancer can be sporadic, familial, or inherited. A strong family history of breast cancer should be explored for ages of breast cancer discovery and any discernable pattern of inheritance to determine whether genetic counseling is appropriate. Smoking, sun exposure, and being overweight are all considered environmental or lifestyle risks for cancer, not an increased genetic risk.

The nurse is assessing a client with chest pain to evaluate whether the client is experiencing angina or myocardial infarction (MI). Which assessment is indicative of an MI? A. Substernal chest discomfort occurring at rest. B. Chest pain brought on by exertion or stress. C. Substernal chest discomfort relieved by nitroglycerin or rest D. Substernal chest pressure relieved only by opioids

D. Substernal chest pressure relieved only by opioids

When caring for a client who receives peritoneal dialysis (PD), which finding does the nurse report to the provider immediately? A. Pulse oximetry reading of 95% B. Sinus bradycardia, rate of 58 beats/min C. Blood pressure of 148/90 mm Hg D. Temperature of 101.2° F (38.4° C)

D. Temperature of 101.2° F (38.4° C)

Which nurse would be assigned to care for a client who is intubated with septic shock due to a methicillin-resistant Staphylococcus aureus (MRSA) infection? A. The LPN/LVN who has 20 years of experience. B. The new RN who recently finished orienting and is working independently with moderately complex clients. C. The RN who will also be caring for a client who had coronary artery bypass graft (CABG) surgery 12 hours ago. D. The RN with 2 years of experience in intensive care unit (ICU).

D. The RN with 2 years of experience in intensive care unit (ICU).

A client begins therapy with lisinopril. What does the nurse consider at the start of therapy with this medication? A. The client's ability to understand medication teaching B. The potential for bradycardia C. Liver function tests D. The risk for hypotension

D. The risk for hypotension Rationale: Angiotensin-converting enzyme (ACE) inhibitors are associated with first-dose hypotension and orthostatic hypotension, which are more likely in those older than 75 years

How will the nurse interpret the finding on a client's pathology report that indicates a cancerous tumor is aneuploid? A. The tumor is completely undifferentiated. B. The tumor is fast growing. C. Metastasis has already occurred. D. The tumor has an abnormal number of chromosomes.

D. The tumor has an abnormal number of chromosomes.

To assess if a client has had a myocardial infarction (MI), which lab value will the nurse assess? A. Creatine kinase-MB fraction (CK-MB) and alkaline phosphatase B. Homocysteine and C-reactive protein C. Total cholesterol, low-density lipoprotein cholesterol, and high-density lipoprotein cholesterol D. Troponin

D. Troponin

What precaution is most important for the nurse to teach the client with systemic lupus erythematosus (SLE) prescribed to take 45 mg of a corticosteroid daily for 2 weeks to manage an SLE flare? Check all your stools for the presence of blood or a black, tarry appearance . Do not suddenly stop taking the drug when your flare is over. Be sure to take this drug with food. Take 30 mg in the morning and 15 mg at night.

Do not suddenly stop taking the drug when your flare is over. All of the precautions are correct and important. However, the most critical precaution is to not suddenly stop taking the drug, which could lead to acute adrenal insufficiency and even death. This dose of the drug (45 mg daily) would need to be tapered down over a period of weeks to prevent adrenal insufficiency.

The nurse is teaching a group of older adult women about the signs and symptoms of urinary tract infection (UTI). What teaching will the nurse include? (Select all that apply.) Dysuria Enuresis Frequency Polyuria Urgency Nocturia

Dysuria Frequency Urgency Nocturia

A client with cognitive impairment has urge incontinence. Which method for achieving continence does the nurse include in the client's care plan? Kegel exercises Habit training Credé method Bladder training

Habit training

The nurse is teaching a group of senior citizens about recommended immunizations. What immunizations would the nurse include? (Select all that apply.) Select all that apply. Herpes zoster vaccine Pneumococcal vaccine polyvalent vaccine Adult Tdap with Td booster every 10 years Annual influenza vaccine Pneumococcal 13-valent conjugate vaccine

Herpes zoster vaccine Pneumococcal vaccine polyvalent vaccine Adult Tdap with Td booster every 10 years Annual influenza vaccine Pneumococcal 13-valent conjugate vaccine All of these immunizations are very important for people over 65 years of age to obtain due to the high risk of the diseases that they help prevent.

Which of the drugs or supplements taken daily taken by a client who is newly diagnosed with drug-induced systemic lupus erythematosus (SLE) does the nurse suspect is most likely to have caused this problem? Vitamin D Lisonopril Aspirin Hydralazine

Hydralazine Hydralazine is a blood pressure medication that has been found to cause drug-induced SLE. None of the other drugs are associated with drug-induced SLE, although lisinopril, an angiotensin-converting enzyme inhibitor, is associated with development of angioedema.

The nurse recognizes that handwashing is the best method for preventing infection. Which action(s) by the Centers for Disease Control (CDC) about hand hygiene are recommended? (Select all that apply.) Select all that apply. If hands are not visibly soiled, use an alcohol-based hand rub. Wash hands before and after wearing gloves. If hands are visibly soiled, wash them with soap and water. Use only soap and water for hand hygiene when planning client contact. Wash hands before performing any invasive client procedure.

If hands are not visibly soiled, use an alcohol-based hand rub. Wash hands before and after wearing gloves. If hands are visibly soiled, wash them with soap and water. Wash hands before performing any invasive client procedure. All of these choices are best practices except for using only soap and water for hand hygiene before client contact. An alcohol-based hand rub is also acceptable for direct or indirect client contact.

The nurse is caring for an older hospitalized client. Which physiologic age-relatedchange(s) increase(s) the client's risk for infection? (Select all that apply.) Select all that apply. Increased cough and gag reflexes Urinary incontinence Decreased intestinal motility Decreased immune response Thinning skin

Increased cough and gag reflexes Urinary incontinence Decreased intestinal motility Older clients have a decreased immune system, decreased intestinal motility, and thinning skin which make them at risk for infection, especially when hospitalized. Urinary incontinence is not a physiologic change of aging; it is a health problem that can be managed. Cough and gag reflexes are decreased rather than increased, which makes older adults at high risk for respiratory infections.

An older adult client diagnosed with urge incontinence is prescribed oxybutynin. Which side effects will the nurse tell the client to expect? (Select all that apply.) Increased intraocular pressure Dry mouth Reddish-orange urine color Constipation Increased blood pressure

Increased intraocular pressure Dry mouth Constipation

After a client is hospitalized for an anaphylactic reaction to a bee sting, a nurse is teaching the client about the use of an epinephrine autoinjector. Which instruction/ instructions should be included in client education? (Select all that apply.) Keep the device with you at all times. After administering the device, hospital monitoring is necessary. Use the device before calling 911. If the drug becomes discolored, order a replacement device. The device CANNOT be given through clothing. Inject the device into your arm or your leg.

Keep the device with you at all times. After administering the device, hospital monitoring is necessary. Use the device before calling 911. If the drug becomes discolored, order a replacement device. Instruct the client to utilize the device at the first symptom of anaphylactic reaction before calling 911. Hospital monitoring is always necessary after utilizing epinephrine for anaphylaxis. The device should be available at all times, as allergens can be encountered in all life situations. For client safety if the drug becomes discolored, it needs to be replaced.The device CAN be given through a thin layer of clothing. The ideal injection site for an epinephrine automatic injector is in the upper thigh.

What is the most important action to prevent harm for the nurse to perform after a client's oral and facial swelling from an angiotensin-converting enzyme inhibitor (ACEI) have resolved? Teaching the client about symptoms to report immediately to the primary health care provider Instructing the client to discard the offending drug after being discharged Monitoring the client for return of symptoms for at least the next 2 to 4 hours Assessing the vein above the IV infusion site for a firm, cordlike texture

Monitoring the client for return of symptoms for at least the next 2 to 4 hours All actions are important, although phlebitis is not likely to occur from IV therapy for angioedema. The ACEI class of drugs have a longer half-life and remain in the body longer than does the corticosteroid infusion used to treat the angioedema. As a result, symptoms can recur after first resolving when corticosteroid therapy is stopped. The client remains at risk and must be monitored for at least 2 to 4 hours for return of angioedema.

Which nursing actions aid in the prevention and early detection of infection in a client at risk? (Select all that apply.) Select all that apply. Obtain cultures as needed. Remove unnecessary medical devices. Monitor the red blood cell (RBC) count. Inspect the skin for coolness and pallor. Promote sufficient nutritional intake. Encourage fluid intake, as appropriate.

Obtain cultures as needed. Remove unnecessary medical devices. Promote sufficient nutritional intake. Promoting sufficient nutritional intake helps prevent and detect early infection in at risk clients. Nutrition has a direct correlation to improvement of general health. Malnutrition, especially protein-calorie malnutrition, places clients at increased risk for infection. Blood cultures would be used to detect a possible systemic infection. Advocating for the removal of unnecessary medical devices (e.g., intravascular or urinary catheters, endotracheal tubes, synthetic implants) may also interfere with normal host defense mechanisms and may help prevent infection.Inspecting the skin does not prevent or detect systemic infections. Fluid intake is important but does not directly relate to prevention or detection of infection. Monitoring the RBC count does not prevent, nor would it detect, infection.

The nurse is performing catheter care. Which nursing action demonstrates proper aseptic technique? Sending a urine specimen to the laboratory for testing Irrigating the catheter daily Positioning the collection bag below the height of the bladder Applying Betadine ointment to the perineal area after catheterization

Positioning the collection bag below the height of the bladder

A 73-year-old woman is admitted to the telemetry unit with Class III left-sided heart failure, type 2 diabetes mellitus and hypertension. When the patient arrives to the unit, the nurse assesses that her color is pale, she is becoming increasingly dyspneic, and appears restless. The patient's husband is very concerned because he thinks his wife is slightly disoriented. The patient has crackles in bilateral lung bases and her oxygen saturation levels are 88% on oxygen at 2 liters per nasal cannula. What actions would most likely achieve the desired outcomes for this client? Which actions should be avoided or are potentially harmful? (Hint: Determine the desired outcomes first to decide which interventions are appropriate and those that should be avoided.)

Prompt identification of the client's decreasing gas exchange is the priority of care. Recognizing that heart failure can lead to pulmonary edema and intervening with early symptoms present is likely to promote desired outcomes. Pulmonary edema is potentially very harmful as this is a life threatening condition.

Which nursing intervention or practice is effective in helping to prevent urinary tract infection (UTI) in hospitalized clients? Recommending that catheters be placed in all clients Encouraging fluid intake Irrigating all catheters daily with sterile saline Reevaluating the need for indwelling catheters

Reevaluating the need for indwelling catheters

A client who is 6 months pregnant comes to the prenatal clinic with a suspected urinary tract infection (UTI). What action will the nurse take with this client? Discharges the client to her home for strict bedrest for the duration of the pregnancy. Instructs the client to drink a minimum of 3 L of fluids daily to "flush out" bacteria. Recommends that the client refrain from having sexual intercourse until after delivery. Refers the client to the clinic nurse practitioner for immediate follow-up.

Refers the client to the clinic nurse practitioner for immediate follow-up.

Which precaution is a priority to prevent harm for the nurse to teach a client with systemic lupus erythematosus (SLE) who is newly prescribed to take hydroxychloroquine for disease management? See your ophthalmologist for visual field testing every 6 months. Report a reduction of joint swelling to your rheumatology health care provider immediately. Report a worsening of joint swelling to your rheumatology health care provider immediately. See your ophthalmologist for intraocular pressure measurement every 6 months.

See your ophthalmologist for visual field testing every 6 months. Hydroxychloroquine has both immunomodulating and anticlotting effects that can be beneficial to clients with SLE. A major complication of this drug is its toxicity to retinal cells causing retinitis that can lead to an irreversible loss of central vision. Clients prescribed hydroxychloroquine are instructed to have frequent eye examinations with visual field testing (before starting the drug and every 6 months thereafter). If retinal toxicity is suspected, the drug is discontinued to preserve the remaining vision.

The nurse is teaching a class about cancer prevention. Which interventions will the nurse include that can prevent bladder cancer? (Select all that apply.) Using pelvic floor muscle exercises Drinking 2½ L of fluid a day Stopping the use of tobacco Wearing a lead apron when working with chemicals Wearing gloves and a mask when working around chemicals and fumes Showering after working with or around chemicals

Stopping the use of tobacco Wearing gloves and a mask when working around chemicals and fumes Showering after working with or around chemicals

Which information does the nurse include when teaching a client about antibiotic therapy for infection? Take antibiotics until symptoms subside, and then stop taking the drugs. Share antibiotics with family members who develop the same infection. Take all antibiotics as prescribed, unless adverse effects develop. Take antibiotics when symptoms of infection develop.

Take all antibiotics as prescribed, unless adverse effects develop. Antibiotics should be taken as prescribed until they are gone. Teach the client about possible side effects and allergic manifestations. The primary health care provider must be contacted immediately if any adverse effects develop.Antibiotics must be taken until they are gone, even if the client feels better or when symptoms of infection appear. They should be taken only by the person for whom they are prescribed and not shared with anyone else.

A client who was treated last month for a severe respiratory infection reports many of the same symptoms today. Which factor in the client's use of antibiotic therapy most likely caused the client's relapse? Taking the antibiotic most days Taking the antibiotic as prescribed Taking the antibiotic before jogging 2 miles daily Taking the antibiotic with a full glass of water

Taking the antibiotic most days Antibiotics not taken as prescribed can result in recurring symptoms, as well as the development of drug-resistant infections and other emerging infections.Taking the antibiotic before jogging is not a contributing factor to the client's relapse. The client who is taking antibiotics as prescribed is not likely to develop recurring symptoms. Taking antibiotics with a full glass of water is a positive action and neither hinders nor promotes antimicrobial therapy.

What is the nurse's interpretation of a laboratory result that indicates a client has a high blood concentration of IgG directed against the human papilloma virus? A) The client is at risk for major hypersensitivity reactions to attenuated vaccines. B) The client is mounting an appropriate response to a recurrent exposure to the virus. C) The client is in the midst of his or her first response to human papilloma infection. D) The client is at increased risk for becoming ill from opportunistic infectious organisms.

The client is mounting an appropriate response to a recurrent exposure to the virus. When naive B-cells become sensitized to a specific microorganism, they divide forming plasma cells and memory cells, both of which retain the antigen sensitization. The plasma cell immediately begins to secrete antibodies in the form of immunoglobulin M (IgM) against the microorganism. Upon later re-exposure to the same antigen, memory cells will secrete immunoglobulin G (IgG) against it. Therefore the presence of high concentrations of specific IgG in the blood indicates a normal immune response to recurrent infection to the same viral infection. Option B is incorrect because IgG does not mediate hypersensitivity reactions. Option A is incorrect because an initial exposure would be indicated by increased IgM levels against the microorganism, not IgG. Option D is incorrect because the presence of high levels of IgG does not indicate a decline in the client's immune status.

A 73-year-old woman is admitted to the telemetry unit with Class III left-sided heart failure, type 2 diabetes mellitus and hypertension. When the patient arrives to the unit, the nurse assesses that her color is pale, she is becoming increasingly dyspneic, and appears restless. The patient's husband is very concerned because he thinks his wife is slightly disoriented. The patient has crackles in bilateral lung bases and her oxygen saturation levels are 88% on oxygen at 2 liters per nasal cannula. What client conditions are consistent with the most relevant information? (Hint: Think about priority collaborative problems that support and contradict the information presented in this situation.)

The client's dyspnea, crackles in the lungs, and decreasing oxygen levels are consistent with class III heart failure. These symptoms support the priority concern of decreased gas exchange. The client's disorientation, pallor, and restlessness suggest a worsening of the client's condition.

A 73-year-old woman is admitted to the telemetry unit with Class III left-sided heart failure, type 2 diabetes mellitus and hypertension. When the patient arrives to the unit, the nurse assesses that her color is pale, she is becoming increasingly dyspneic, and appears restless. The patient's husband is very concerned because he thinks his wife is slightly disoriented. The patient has crackles in bilateral lung bases and her oxygen saturation levels are 88% on oxygen at 2 liters per nasal cannula. Which possibilities or explanations are most likely to be present in this client situation? Which possibilities or explanations are the most serious? (Hint: Consider all possibilities and determine their urgency and risk for this client.)

The client's symptoms could indicate pulmonary edema which is life-threatening. The husband's concern that his wife is disoriented is important as disorientation can be an early sign in older adults of pulmonary edema.

Which statement(s) regarding type IV hypersensitivity reactions is/are true? (Select all that apply.) The major mechanism of the reaction is the release of mediators from sensitized T-cells that trigger antigen destruction by macrophages. Type IV responses are usually directed against non-self but the response is excessive. The second phase of the reaction with accumulation of excess bradykinin is responsible for development of angioedema. The secondary phase, when prolonged, is primarily responsible for autoimmune disorders. Rashes and blister formation from poison ivy exposure are a typical response for this type of hypersensitivity reaction. Antihistamines are of minimal benefit because the reactions are mediated by IgE rather than histamine.

The major mechanism of the reaction is the release of mediators from sensitized T-cells that trigger antigen destruction by macrophages. Type IV responses are usually directed against non-self but the response is excessive. Rashes and blister formation from poison ivy exposure are a typical response for this type of hypersensitivity reaction. Antihistamines are of minimal benefit because the reactions are mediated by IgE rather than histamine. Type IV delayed hypersensitivity reactions have T-lymphocytes (T-cells) as the activated immune system component triggering the excessive responses. A classic example is allergy to poison ivy.Sensitized T-cells (from a previous exposure) respond to an antigen by releasing chemical mediators and triggering macrophages to destroy the antigen; however, histamine is not one of the mediators, making antihistamines of minimal benefit.A type IV response with edema, induration, ischemia, and tissue damage at the site of the exposure typically occurs hours to days after exposure.Angioedema is a type I response, not a type IV response.

A 73-year-old woman is admitted to the telemetry unit with Class III left-sided heart failure, type 2 diabetes mellitus and hypertension. When the patient arrives to the unit, the nurse assesses that her color is pale, she is becoming increasingly dyspneic, and appears restless. The patient's husband is very concerned because he thinks his wife is slightly disoriented. The patient has crackles in bilateral lung bases and her oxygen saturation levels are 88% on oxygen at 2 liters per nasal cannula. What assessment information in this client situation is the most important and immediate concern for the nurse? (Hint: Identify the relevant information first to determine what is most important.)

The most relevant information is the client's history of heart failure, the increasing dyspnea, restlessness, disorientation, oxygen saturation and bilateral crackles.

A 73-year-old woman is admitted to the telemetry unit with Class III left-sided heart failure, type 2 diabetes mellitus and hypertension. When the patient arrives to the unit, the nurse assesses that her color is pale, she is becoming increasingly dyspneic, and appears restless. The patient's husband is very concerned because he thinks his wife is slightly disoriented. The patient has crackles in bilateral lung bases and her oxygen saturation levels are 88% on oxygen at 2 liters per nasal cannula. Which actions are the most appropriate and how should they be implemented? In what priority order should they be implemented? (Hint: Consider health teaching, documentation, requested health care provider orders or prescriptions, nursing skills, collaboration with or referral to health team members, etc.)

The nurse will first set the client up in a high Fowler's position to maximize lung expansion and decrease venous return to the heart. Then the nurse will administer oxygen as directed to maintain an oxygen saturation of at least 90%. The nurse will notify the primary care provider and collaboration with the respiratory therapist. The healthcare provider will likely prescribe rapid-acting diuretics such as furosemide. Pulmonary edema can progress rapidly and mechanical ventilation may be required for some clients.

The nurse is preparing to draw blood from a client receiving a course of vancomycin about 30 minutes before the next scheduled dose. For what laboratory test would the blood specimen be most likely tested? Trough drug level Blood culture and sensitivity White blood cell (WBC) count Peak drug level

Trough drug level When clients receive some intravenous antibiotics, it is essential that the levels of the drug stay consistent within a therapeutic range. To determine if that is the case, peak and trough levels are drawn. A trough level indicates the lowest level of drug available in the blood and is drawn shortly before the next scheduled drug dose. A peak level is assessed 30 to 60 minutes after the drug is given. A culture and sensitivity would not be done while the client is on antibiotics. The WBC count should be decreasing as a result of antibiotic therapy.

Which type of hypersensitivity reaction will the nurse suspect in a client who develops as circular rash on the skin underneath a new necklace worn for 3 days? Type IV Type I Type II Type III

Type IV A type IV delayed hypersensitivity reaction occurs when sensitized T-cells respond to an antigen by releasing chemical mediators and triggering macrophages. This reaction causes a rash as seen in a metal allergy exposure.A type I reaction occurs rapidly after exposure and is mediated by immunoglobulin E (IgE). Type II reactions occur when the body makes autoantibodies directed against self-cells and attack those cells. Type III reactions occur when an abundance of immune complexes are made and they get stuck in small vessels causing inflammation.

A client diagnosed with urge incontinence is started on tolterodine. What interventions will the nurse suggest to alleviate the side effects of this drug? (Select all that apply.) Limit the intake of dairy products. Use hard candy for dry mouth. Encourage increased fluids. Increase fiber intake. Take the drug at bedtime.

Use hard candy for dry mouth. Encourage increased fluids. Increase fiber intake.

A client is referred to a home health agency after being hospitalized with overflow incontinence and a urinary tract infection. Which nursing action can the home health RN delegate to the home health aide (assistive personnel [AP])? Using a bladder scanner to check residual bladder volume after the client voids Inserting a straight catheter as necessary if the client is unable to empty the bladder Teaching the client how to use the Credé maneuver to empty the bladder more fully Assisting the client in developing a schedule for when to take prescribed antibiotics

Using a bladder scanner to check residual bladder volume after the client voids

While in the hospital, a client developed a methicillin-resistant infection in an open foot ulcer. Which nursing action would be appropriate for this client? Wear a gown and gloves to prevent contact with the client or client-contaminated items. Have the client wear a surgical mask when being transported out of the room. Wear a mask when working within 3 feet (91 cm) of the client. Assign the client to a private room with a negative airflow.

Wear a gown and gloves to prevent contact with the client or client-contaminated items. Caregivers should wear a gown and gloves to prevent contact with the client or contaminated items when caring for a client with this infection. This is the best way to prevent the spread of infection. Gloves should also be worn when entering the room.The client does not require a private room or respiratory isolation, and does not need to wear a surgical mask when being transported out of the room because the infection is not airborne. Use of a mask is not the best way to prevent the spread of this infection.

Which precaution is appropriate for the nurse to take to prevent the transmission of Clostridium difficile infection? Carefully wash hands that are visibly soiled. Wear a mask with eye protection and perform proper handwashing. Wear gloves when in contact with the client's body secretions or fluids. Wear a mask and gloves when in contact with the client.

Wear gloves when in contact with the client's body secretions or fluids. The nurse must wear gloves and wash hands before and after potential exposure to the client's body secretions or fluids. C. difficile infection requires Contact Precautions. Hands must be properly washed before and after any contact with the client with C. difficile infection. Alcohol-based hand rubs are not effective for hand hygiene in the care of clients with C. difficile.Hands must be washed even if not visibly soiled. It is not necessary to wear a mask when caring for clients with C. difficile infection. A mask and eye protection are not necessary to prevent transmission of C. difficile.

An older adult woman who reports a change in bladder function says, "I feel like a child who sometimes pees her pants." What is the best nursing response? a) "Have you tried using the toilet every couple of hours?" b) "How does that make you feel?" c) "We can fix that." d) "That happens when we get older."

a) "Have you tried using the toilet every couple of hours?"

The nurse is caring for client who has just returned from the operating room for cystoscopy performed under conscious sedation. Which assessment finding requires immediate nursing action? a) Temperature of 100.8° F (38.2° C) b) Lethargy c) Pink-tinged urine d) Urinary frequency

a) Temperature of 100.8° F (38.2° C)

The nurse is teaching a client with peripheral arterial disease. What teaching will the nurse include? a) "Walk to the point of leg pain, then rest, resuming when pain stops." b) "Inspect your legs daily for brownish discoloration around the ankles." c) "Apply a heating pad to the legs if they feel cold." d) "Elevate your legs above heart level to prevent swelling."

a) "Walk to the point of leg pain, then rest, resuming when pain stops." Rationale: The teaching point the nurse include for a client with PAD is walk to the point of leg pain, rest, and then resume when pain stops. Exercise may improve arterial blood flow by building collateral circulation. Instruct the client to walk until the point of claudication, stop and rest, and then walk a little farther.Elevating the legs in PAD decreases blood flow and increases ischemia. Brown discoloration around the ankles is characteristic of venous disease. Application of heat must be avoided in clients with PAD due to a lack of sensation and possible burns to the legs.

The client who wants to use Truvada for preexposure prophylaxis (PreP) asks the nurse why testing is needed for HIV status before starting this drug. How does the nurse respond? a) "although this drug can help prevent HIV infection, it is not enough by itself to control "the disease if you are HIV positive." b) "the side effects of this drug are worse if you have a detectable HIV viral load." c) "if you take this drug and are HIV positive, your risk for co-infection with the hepatitis B virus is increased." d) some people have a genetic mutation that increases the risk for life-threatening reactions "while taking this drug if they are also HIV positive."

a) "although this drug can help prevent HIV infection, it is not enough by itself to control "the disease if you are HIV positive." b) "the side effects of this drug are worse if you have a detectable HIV viral load."

The nurse is using a bladder scanner on a female client to estimate bladder volume. Which action will the nurse take? (Select all that apply.) a) Aim the scanner toward the client's coccyx to visualize the bladder. b) Select the female icon since the client has had a hysterectomy. c) Two readings should be completed for best accuracy. d) Gently insert the scanner probe into the vagina. e) Place a gel pad over the client's pubic area.

a) Aim the scanner toward the client's coccyx to visualize the bladder. c) Two readings should be completed for best accuracy. e) Place a gel pad over the client's pubic area.

For a client with an 8-cm abdominal aortic aneurysm, which assessment data must be addressed immediately? a) Blood pressure (BP) 192/102 mm Hg b) Report of constipation c) Anxiety d) Heart rate 52 beats/min

a) Blood pressure (BP) 192/102 mm Hg Rationale: The problem that must be addressed immediately in a client with an 8-cm abdominal aneurysm is a BP of 192/102 mm Hg. Elevated blood pressure can increase the rate of aneurysmal enlargement and risk for early rupture.The nurse must consider the client's usual pulse. However, bradycardia does not pose a risk for aneurysm rupture. Straining at stool can elevate blood pressure and pose a risk for dissection. However, a potential problem would not be addressed before an actual problem. Anxiety may be benign or may be a symptom of something serious. However, the elevated blood pressure is an immediate risk.

The nurse is caring for the following clients who are scheduled for a computed tomography (CT) scan with contrast. For which clients will the nurse communicate safety concerns to the health care provider (HCP)? (Select all that apply.) a) Client who took metformin 4 hours ago b) Client with an allergy to shrimp c) Client who requests morphine sulfate every 3 hours d) Client with a history of asthma e) Client with a blood urea nitrogen of 62 mg/dL (22.1 mmol/L) and a creatinine of 2.0 mg/dL (177 umol/L)

a) Client who took metformin 4 hours ago b) Client with an allergy to shrimp d) Client with a history of asthma e) Client with a blood urea nitrogen of 62 mg/dL (22.1 mmol/L) and a creatinine of 2.0 mg/dL (177 umol/L)

The charge nurse is making client assignments for the day shift. Which client is best to assign to an LPN/LVN? a) Client with polycystic kidney disease who is having a kidney ultrasound. b) Client with glomerulonephritis who is having a kidney biopsy. c) Client who is going for a cystoscopy and cystourethroscopy. d) Client who has just returned from having a kidney artery angioplasty.

a) Client with polycystic kidney disease who is having a kidney ultrasound.

Which laboratory test will the nurse assess as the best indicator of kidney function? a) Creatinine b) Blood urea nitrogen (BUN) c) Aspartate aminotransferase (AST) d) Alkaline phosphatase

a) Creatinine

When performing bladder scanning to detect residual urine in a female client, the nurse must first assess which factor? a) History of hysterectomy b) Abdominal girth c) Hematuria d) Presence of urinary infection

a) History of hysterectomy

The nurse is reviewing the medical record for a client with polycystic kidney disease who is scheduled for computed tomographic angiography with contrast: Which nursing intervention is essential? a) Hold the metformin 24 hours before and on the day of the procedure. b) Notify the provider regarding blood glucose and glycosylated hemoglobin (HbA1c) values. c) Report the blood urea nitrogen (BUN) and creatinine. d) Obtain a thyroid-stimulating hormone (TSH) level.

a) Hold the metformin 24 hours before and on the day of the procedure.

Which client assessment data indicates to the nurse that the client has a potential need for fluids? a) Increased blood urea nitrogen b) Increased creatinine c) Decreased sodium d) Pale-colored urine

a) Increased blood urea nitrogen

The nurse is caring for a client with uremia. What assessment data will the nurse anticipate? a) Nausea and vomiting b) Insomnia c) Cyanosis of the skin d) Tenderness at the costovertebral angle (CVA)

a) Nausea and vomiting

A client is scheduled for a cystoscopy later this morning. The consent form is not signed, and the client has not had any preoperative medication. The nurse notes that the health care provider (HCP) visited the client the day before. What action will the nurse take? a) Notifies the department and the HCP. b) Asks the client's spouse to sign the form. c) Cancels the procedure. d) Asks the client to sign the informed consent.

a) Notifies the department and the HCP.

A client had a computed tomography (CT) scan with contrast dye 8 hours ago. Which nursing intervention is the priority for this client? a) Promoting fluid intake b) Medicating for pain c) Monitoring for hematuria d) Maintaining bedrest

a) Promoting fluid intake

The nurse is assigned to all of these clients. Which client would the nurse assess first? a) The client who had percutaneous vascular intervention of the right femoral artery 30 minutes ago. b) The client admitted with hypertensive crisis who has a nitroprusside drip and blood pressure of 149/80 mm Hg. c) The client with peripheral vascular disease who has a left leg ulcer draining purulent yellow fluid. d) The client who had a right femoral-popliteal bypass 3 days ago and has ongoing edema of the foot.

a) The client who had percutaneous vascular intervention of the right femoral artery 30 minutes ago. Rationale: The client who would be assessed first is the client who had a percutaneous vascular intervention of the right femoral artery 30 minutes ago. This client must have checks of vascular status and vital signs every 15 minutes in the first hour after the procedure.The client admitted with hypertensive crisis has stabilized and is not in need of immediate assessment. The client with peripheral vascular disease is the most stable and can be seen last. The client who had a right femoral-popliteal bypass is not in need of immediate assessment and can be assessed after the client who had a percutaneous vascular intervention.

The nurse is caring for a client who had abdominal aortic aneurysm (AAA) repair. Which assessment data is most concerning to the nurse? a) Urine output of 20 mL over 2 hours b) Blood pressure of 106/58 mm Hg c) +3 pedal pulses d) Absent bowel sounds

a) Urine output of 20 mL over 2 hours Rationale: The nurse caring for a client who had an AAA repair would be most alarmed with the client's urine output of 20 mL over 2 hours. Complications post AAA stent repair include bleeding, which may manifest as signs of hypovolemia and oliguria.Reduction of systolic blood pressure to 100 to 120 mm Hg is appropriate. Paralytic ileus may be a complication of AAA repair, but is not a priority over decreased urine output. +3 pedal pulses is a normal physical assessment finding.

Which statements about the transmission of HIV are true? (Select all that apply.) a) clients with HIV-III and no drug therapy are very infectious. b) even with appropriate drug therapy, most clients infected with HIV live only about 5 years after diagnosis. c) HIV may be transmitted only during the end stages of the disease. d) the most common transmission route is casual contact. e) newly infected clients with a high viral load are very infectious. f) HIV-positive clients who have an undetectable viral load appear to not transmit the disease.

a) clients with HIV-III and no drug therapy are very infectious. e) newly infected clients with a high viral load are very infectious. f) HIV-positive clients who have an undetectable viral load appear to not transmit the disease.

Which signs and symptoms does the nurse expect to find in a client diagnosed with Pneumocystis jiroveciinfection? a) dyspnea, tachypnea, persistent dry cough, and fever b) substernal chest pain and difficulty swallowing c) fever, persistent cough, and vomiting blood d) cough with copious thick sputum, fever, and dyspnea

a) dyspnea, tachypnea, persistent dry cough, and fever

Which laboratory results does the nurse expect to decrease in a client who has untreated HIV-III (AIDS)? (Select all that apply.) a) total white blood cell count b) viral load c) CD8+ T-cell d) HIV antibodies e) CD4+ T-cell f) lymphocytes

a) total white blood cell count e) CD4+ T-cell f) lymphocytes

Which conditions or factors will the nurse teach at a community seminar as probable transmission routes for HIV? (Select all that apply.) a) using injection drugs b) sitting on public toilets c) changing a diaper on an HIV positive child d) having unprotected intercourse with multiple partners e) breast-feeding f) being bitten by mosquitos

a) using injection drugs d) having unprotected intercourse with multiple partners e) breast-feeding

Which laboratory test does the nurse analyze to determine the effectiveness of combination antiretroviral drug therapy in an HIV-positive client? a) viral load testing b) enzyme-linked immunosorbent assay c) fourth generation testing d) western blot analysis

a) viral load testing

A client with chronic kidney disease asks the nurse about the relationship between the disease and high blood pressure. What is the nurse's best response? a. "Because the kidneys cannot get rid of fluid, blood pressure goes up." b. "The damaged kidneys no longer release a hormone that prevents high blood pressure." c. "The waste products in the blood interfere with other mechanisms that control blood pressure." d. "This is a compensatory mechanism that increases blood flow through the kidneys in an effort to get rid of some of the waste products."

a. "Because the kidneys cannot get rid of fluid, blood pressure goes up." The nurse's best response to a client with chronic kidney disease and high blood pressure is, "Because the kidneys cannot get rid of fluid, blood pressure goes up." In chronic kidney disease, fluid levels increase in the circulatory system.The statements asserting that damaged kidneys no longer release a hormone to prevent high blood pressure, waste products in the blood interfere with other mechanisms controlling blood pressure, and high blood pressure is a compensatory mechanism that increases blood flow through the kidneys in attempt excrete waste products are not accurate regarding the relationship between chronic kidney disease and high blood pressure.

A client, who is a mother of two, has autosomal dominant polycystic kidney disease (ADPKD). Which statement by the client indicates a need for further education about her disease? a. "By maintaining a low-salt diet in our house, I can prevent ADPKD in my children." b. "Even though my children don't have symptoms at the same age I did, they can still have ADPKD." c. "If my children have the ADPKD gene, they will have cysts by the age of 30." d. "My children have a 50% chance of inheriting the ADPKD gene that causes the disease."

a. "By maintaining a low-salt diet in our house, I can prevent ADPKD in my children." Further teaching about ADPKD when a mother of two says, "By maintaining a low-salt diet in our house, I can prevent ADPKD in my children." There is no way to prevent ADPKD, although early detection and management of hypertension may slow the progression of kidney damage. Limiting salt intake can help control blood pressure.Limiting salt intake can help control blood pressure. Presentation of ADPKD can vary by age of onset, manifestations, and illness severity, even in one family. Almost 100% of those who inherit a polycystic kidney disease (PKD) gene will develop kidney cysts by age 30. Children of parents who have the autosomal dominant form of PKD have a 50% chance of inheriting the gene that causes the disease.

What is the first action a nurse should take after sustaining a needlestick injury after injecting a client who is known to be HIV positive? a) send the syringe and needle to the laboratory for analysis of viral load. b) inform the charge nurse. c) thoroughly scrub and flush the puncture site. d) go to the employee clinic for postexposure prophylaxis.

c) thoroughly scrub and flush the puncture site.

The school nurse is counseling a teenage student about how to prevent kidney trauma. Which statement by the student indicates a need for further teaching? a. "I can't play any type of contact sports because my brother had kidney cancer." b. "I avoid riding motorcycles." c. "I always wear pads when playing football." d. "I always wear a seat belt in the car."

a. "I can't play any type of contact sports because my brother had kidney cancer." Further teaching about preventing kidney trauma is needed when the teenage student says, "I can't play any type of contact sports because my brother had kidney cancer." Contact sports and high-risk activities must be avoided if a person has only one kidney. A family history of kidney cancer does not prohibit this type of activity.To prevent kidney and genitourinary trauma, caution would be taken when riding bicycles and motorcycles. People need to wear appropriate protective clothing when participating in contact sports. Anyone riding in a car must wear a seat belt.

When caring for a client with nephrotic syndrome, which intervention would be included in the plan of care? a. Administering angiotensin-converting enzyme (ACE) inhibitors to decrease protein loss b. Administering heparin to prevent deep vein thrombosis (DVT) c. Providing antibiotics to decrease infection d. Providing transfusion of clotting factors

a. Administering angiotensin-converting enzyme (ACE) inhibitors to decrease protein loss ACE inhibitors need to be included in a plan of care for a client with nephrotic syndrome. ACE inhibitors can decrease protein loss in the urine.Heparin is administered for DVT, but in nephrotic syndrome it may reduce vascular defects and improve kidney function. Glomerulonephritis may occur secondary to an infection, but it is an inflammatory process. Antibiotics are not indicated for nephrotic syndrome. Clotting factors are not indicated unless bleeding and coagulopathy are present.

The nurse is caring for a client who has just returned to the surgical unit after a radical nephrectomy. Which assessment information alarms the nurse? a. Blood pressure is 98/56 mm Hg; heart rate is 118 beats/min. b. Urine output over the past hour was 80 mL. c. Pain is at a level 4 (on a 0-to-10 scale). d. Dressing has a 1-cm area of bleeding.

a. Blood pressure is 98/56 mm Hg; heart rate is 118 beats/min. A blood pressure of 98/56 mm Hg, and a heart rate of 118 beats/min in a client who just returned to the unit after a radical nephrectomy, alarms the nurse. Bleeding is a complication of radical nephrectomy. Tachycardia and hypotension may indicate impending hypovolemic or hemorrhagic shock. The surgeon must be notified immediately and fluids must be administered, complete blood count needs to be checked, and blood administered, if necessary.A urine output of 80 mL can be considered normal. The nurse can administer pain medication but must address hemodynamic instability and possible hemorrhage first. Administering pain medication to a client who has developed shock will exacerbate hypotension. A dressing with a 1-cm area of bleeding is expected postoperatively.

When caring for a client 24 hours after a nephrectomy, the nurse notes that the client's abdomen is distended. Which action does the nurse perform next? a. Check vital signs. b. Notify the surgeon. c. Continue to monitor. d. Insert a nasogastric (NG) tube.

a. Check vital signs. After noting a distended abdomen in a client who had a nephrectomy 24 hours ago, the nurse next needs to check the client's vital signs. The client's abdomen may be distended from bleeding. Hemorrhage or adrenal insufficiency causes hypotension, so vital signs must be taken to see if a change in blood pressure has occurred.The surgeon would be notified after vital signs are assessed. Just continuing to monitor is not appropriate. An NG tube is not indicated for this client.

Which assessment findings does the nurse expect in a client with kidney cancer? (Select all that apply.) a. Erythrocytosis b. Hypokalemia c. Hypercalcemia d. Hepatic dysfunction e. Increased sedimentation rate

a. Erythrocytosis c. Hypercalcemia d. Hepatic dysfunction e. Increased sedimentation rate Assessment findings the nurse expects to assess in a client with kidney cancer include: erythrocytosis, hypercalcemia, hepatic dysfunction, and increased sedimentation rate. Erythrocytosis alternating with anemia and hepatic dysfunction with elevated liver enzymes may occur with kidney cancer. Parathyroid hormone produced by tumor cells can cause hypercalcemia. An elevation in sedimentation rate may occur in paraneoplastic syndromes.Potassium levels (hypokalemia) are not altered in kidney cancer.

When caring for a client with polycystic kidney disease, which goal is most important? a. Preventing progression of the disease b. Performing genetic testing c. Assessing for related causes d. Consulting with the dialysis unit

a. Preventing progression of the disease The most important goal for a client with polycystic kidney disease is preventing complications and progression of the disease.Genetic testing would be done, but this is not a priority. Assessment for related causes is an intervention, not a goal. Not all clients with polycystic kidney disease require dialysis.

When taking the health history of a client with acute glomerulonephritis (GN), the nurse questions the client about which related cause of the problem? a. Recent respiratory infection b. Hypertension c. Unexplained weight loss d. Neoplastic disease

a. Recent respiratory infection For the client with acute glomerulonephritis, the nurse questions the client about any recent respiratory infection. An infection often occurs before the kidney manifestations of acute GN. The onset of symptoms is about 10 days from the time of infection.Hypertension is a result of glomerulonephritis, not a cause. Weight gain, not weight loss, is symptomatic of fluid retention in GN. Cancers are not part of the cause of GN.

Which condition may predispose a client to chronic pyelonephritis? a. Spinal cord injury b. Cardiomyopathy c. Hepatic failure d. Glomerulonephritis

a. Spinal cord injury The client with a spinal cord injury may be predisposed to chronic pyelonephritis. Chronic pyelonephritis usually occurs with structural deformities, urinary stasis, obstruction, or reflux. Reduced bladder tone from spinal cord injury contributes to stasis and reflux. Conditions that lead to urinary stasis include prolonged bedrest and paralysis. Obstruction can be caused by stones, kidney cancer, scarring from pelvic radiotherapy or surgery, recurrent infection, or injury.Cardiomyopathy or weakness of the heart muscle may cause kidney impairment, not an infection. Pyelonephritis may damage the kidney, not the liver. Glomerulonephritis may result from infection but may not cause infection of the kidney.

The nurse is teaching a client who needs a clean-catch urine specimen. What teaching will the nurse include? "Save all urine for 24 hours." b) "Do not touch the inside of the container." c) "Use the sponges to cleanse the urethra, and then initiate voiding directly into the cup." d) "You will receive an isotope injection, then I will collect your urine."

b) "Do not touch the inside of the container."

The nurse is teaching a client how to provide a clean-catch urine specimen. Which client statement indicates that teaching was effective? a) "I will have to drink 2 L of fluid before providing the sample." b) "I'll start to urinate in the toilet, stop, and then urinate into the cup." c) "It is best to provide the sample while I am bathing." d) "I must clean with the wipes and then urinate directly into the cup."

b) "I'll start to urinate in the toilet, stop, and then urinate into the cup."

Which statement made to the nurse by an assistive personnel (AP) assigned to care for an HIV-positive client indicates a breach of confidentiality and requires further education by the nurse? a) "the client's spouse told me she got HIV from a blood transfusion." b) "the other assistive personnel and I were out in the hallway discussing our concern about getting HIV from our client." c) "i told family members they need to wash their hands when they enter and leave the room." d) "yes, I understand the reasons why I have don't need to wear gloves when I feed the client."

b) "the other assistive personnel and I were out in the hallway discussing our concern about getting HIV from our client."

The nurse is caring for a client with dark-colored toe ulcers and blood pressure (BP) of 190/100 mm Hg. Which nursing action does the nurse delegate to the LPN/LVN? a) Obtain a request from the primary health care provider for a dietary consult. b) Administer a clonidine patch for hypertension. c) Develop a plan for discharge, and assess home care needs. d) Assess leg ulcers for signs of infection.

b) Administer a clonidine patch for hypertension. Rationale: The action the nurse delegates to the LPN/LVN caring for a client with dark-colored toe ulcers and a BP of 190/100 mm Hg is to administer a clonidine patch for hypertension. Administering medication is within the scope of practice for the LPN/LVN.The RN is responsible for physical assessments, making referrals for other services, and developing the plan of care for the hospitalized client.

The nurse is assessing a 54-year-old male client for risk of atherosclerosis. What assessment data is associated with an increase in risk? (Select all that apply.) a) Takes acetylsalicylic acid daily. b) BMI is 32. c) History of type 2 diabetes mellitus. d) LDL of 160 mg/dL. e) The client's father has lung cancer. f) Current smoking history.

b) BMI is 32. c) History of type 2 diabetes mellitus. d) LDL of 160 mg/dL. f) Current smoking history. Rationale: Risk factors that contribute to atherosclerosis include: an increase in LDL (160 mg/dL is high), obesity (as indicated by a BMI is 32), smoking, and type 2 diabetes.ASA is used as prophylaxis for atherosclerotic disease/coronary artery disease to prevent platelet adhesion. While atherosclerosis can be genetic, the fact that the client's father has lung cancer does not increase his risk for atherosclerosis.

The nurse is caring for a client who is being treated for hypertensive crisis. Which prescribed medication would the nurse question? a) Enalapril b) Dopamine c) Labetalol d) Sodium nitroprusside

b) Dopamine Rationale: The nurse would question the prescription for dopamine. Dopamine is used for its inotropic and vasoconstrictive properties to raise blood pressure, and would not be used in hypertensive crisis.Enalapril, an angiotensin-converting enzyme inhibitor, may be used intravenously in hypertensive emergencies. Sodium nitroprusside, a direct-acting vasodilator, may be used intravenously to lower blood pressure quickly in hypertensive emergencies. Labetalol, an intravenous calcium channel blocker, is used in hypertensive crisis when oral therapy is not feasible.

The nurse is teaching a class about kidney and urinary changes that occur with age. What teaching will the nurse include? (Select all that apply.) a) Drug clearance is often increased which produces more drug reactions. b) Glomerular filtration rate decreases which increases the risk for fluid overload. c) Urinary sphincters lose tone and weaken with age. d) Blood flow to the kidneys increases promoting nocturia. e) The ability to concentrate urine decreases which creates urgency.

b) Glomerular filtration rate decreases which increases the risk for fluid overload. c) Urinary sphincters lose tone and weaken with age. e) The ability to concentrate urine decreases which creates urgency.

The nurse is teaching the client dietary methods to reduce LDL levels. What teaching will the nurse include? (Select all that apply.) a) Aim for 10% of calories from saturated fat b) Limit trans-fat intake. c) Emphasize the intake of whole grains. d) Avoid cooking with all oil. e) Nuts are a good snack food. f) Try to purchase skinless chicken to cook with.

b) Limit trans-fat intake. c) Emphasize the intake of whole grains. e) Nuts are a good snack food. f) Try to purchase skinless chicken to cook with. Rationale: The American Heart Association publishes dietary recommendations to decrease LDL levels. These recommendations include: emphasizing the intake of whole grains, vegetables, and fruits; consuming poultry without the skin; consuming low-fat dairy products and nuts; cooking with nontropical oils (e.g. Canola); limiting trans-fat intake and aiming for a dietary pattern that includes 5% to 6% of calories from saturated fat

The nurse is assessing a client with arterial insufficiency. What assessment data would cause the nurse to suspect an acute arterial occlusion of the right lower extremity? (Select all that apply.) a) Tachycardia b) Mottling of right foot and lower leg c) Bounding right pedal pulses d) Numbness and tingling of right foot e) Hypertension f) Cold right foot

b) Mottling of right foot and lower leg d) Numbness and tingling of right foot f) Cold right foot Rationale: Signs/symptoms of acute arterial occlusion of the right lower extremity include cold right foot, numbness and tingling of the right foot, and mottling and tingling of the right foot. Pain, pallor, pulselessness, paresthesia, paralysis, poikilothermia (cool limb), and mottled color are characteristics of acute arterial occlusion.Hypertension presents risk for atherosclerosis, but not for acute arterial occlusion. The pulse rate does not indicate occlusion, but rather quality. Absence of pulse, rather than bounding pulse, is a symptom of acute arterial occlusion.

Which assessment by a new nurse requires the charge nurse to intervene? a) Assessing pedal pulses by Doppler b) Simultaneously palpating bilateral carotids c) Measuring blood pressure in both arms d) Measuring capillary refill in the fingertips

b) Simultaneously palpating bilateral carotids Rationale: The vascular assessment by the new nurse that requires intervention by the charge nurse is simultaneously palpating bilateral carotids. Carotid arteries are palpated separately because of the risk for inadequate cerebral perfusion and the risk for causing the client to faint.Prolonged capillary filling generally indicates poor circulation, and is an appropriate assessment. Many clients with vascular disease have poor blood flow. Pulses that are not palpable may be heard with a Doppler probe. Because of the high incidence of hypertension in clients with atherosclerosis, blood pressure is often assessed in both arms.

A new nurse is caring for four clients. Which client is at risk for secondary hypertension? a) The client who is physically inactive. b) The client with kidney disease. c) The client with depression. d) The client who eats a high-sodium diet.

b) The client with kidney disease. Rationale: The client who is most at risk for secondary hypertension is the client with kidney disease. Kidney disease is one of the most common causes of secondary hypertension.Some psychiatric conditions can exacerbate essential hypertension, but secondary hypertension is caused by a disease process or drugs. High-sodium intake is a risk factor for essential hypertension, not for secondary hypertension, which is caused by disease states or medications. Physical inactivity is a risk factor for essential hypertension.

With which antiretroviral drug class will the nurse teach clients to prevent harm by reporting any new onset muscle weakness and muscle pain to the immunity health care provider? a) fusion inhibitors b) integrase inhibitors c) nucleoside reverse transcriptase inhibitors d) protease inhibitors

b) integrase inhibitors

A client is hesitant to talk to the nurse about genitourinary dysfunction symptoms. What is the nurse's best response? a. "Don't worry, no one else will know." b. "Take your time. What is bothering you the most?" c. "Why are you hesitant?" d. "You need to tell me so we can determine what is wrong."

b. "Take your time. What is bothering you the most?" The nurse's best response when a client is hesitant to talk about genitourinary dysfunction is "take your time. What is bothering you the most?" Asking the client what is bothering him or her expresses patience and understanding when trying to identify the client's problem. It is important for the nurse to encourage the client to tell his/her own story in familiar, comfortable language.Telling the client that others will not know is untrue because the client's symptoms will be in the medical record for other health care personnel to see. Asking why the client is hesitant can seem accusatory and threatening to the client. Admonishing the client to disclose his or her symptoms is too demanding; the nurse must be more understanding of the client's embarrassment.

What is the appropriate range of urine output for the client who has just undergone a nephrectomy? a. 23 to 30 mL/hr b. 30 to 50 mL/hr c. 41 to 60 mL/hr d. 50 to 70 mL/hr

b. 30 to 50 mL/hr A urine output of 30 to 50 mL/hr or 0.5 to 1 mL/kg/hr is considered within acceptable range for the client who is post nephrectomy.Output of less than 25 to 30 mL/hr suggests decreased blood flow to the remaining kidney and the onset or worsening of acute kidney injury. A large urine output, followed by hypotension and oliguria, is a sign of adrenal insufficiency.

The nurse anticipates that a client who develops hypotension and oliguria post nephrectomy may need the addition of which element to the regimen? a. Increase in analgesics b. Addition of a corticosteroid c. Administration of a diuretic d. Course of antibiotic therapy

b. Addition of a corticosteroid The nurse expects that the post nephrectomy client with hypotension and oliguria will need the addition of corticosteroids. Loss of water and sodium occurs in clients with adrenal insufficiency, which is followed by hypotension and oliguria. So corticosteroids may be needed.The nurse must use caution when administering analgesics to a hypotensive client. No indication suggests that pain is present in this client. A diuretic would further contribute to fluid loss and hypotension, potentially worsening kidney function. A few doses of antibiotics are used prophylactically preoperatively and postoperatively. Additional therapy is used when evidence of infection exists.

The RN is working with unlicensed assistive personnel (UAP) in caring for a group of clients. Which action is best for the RN to delegate to UAP? a. Assessing the vital signs of a client who was just admitted with blunt flank trauma and hematuria b. Assisting a client who had a radical nephrectomy 2 days ago to turn in bed c. Helping the primary health care provider with a kidney biopsy for a client admitted with acute glomerulonephritis d. Palpating for bladder distention on a client recently admitted with a ureteral stricture

b. Assisting a client who had a radical nephrectomy 2 days ago to turn in bed The best action for the RN is to have the UAP assist a client who had a radical nephrectomy 2 days ago to turn in bed. A UAP would be working within legal guidelines when assisting a client to turn in bed.Although assessment of vital signs is within the scope of practice for UAP, the trauma victim would be assessed by the RN because interpretation of the vital signs is needed. Assisting with procedures such as kidney biopsy and assessment for bladder distention are responsibilities of the professional nurse that would not be delegated to staff members with a limited scope of education.

When assessing a client with acute glomerulonephritis, which finding causes the nurse to notify the primary health care provider? a. Purulent wound on the leg b. Crackles throughout the lung fields c. History of diabetes d. Cola-colored urine

b. Crackles throughout the lung fields The nurse notifies the primary health care provider if crackles throughout the lung fields are heard in a client with acute glomerulonephritis. Crackles indicate fluid overload resulting from kidney damage. Shortness of breath and dyspnea are typically associated. The primary health care provider must be notified of this finding.Glomerulonephritis may result from infection (e.g., purulent wound); it is not an emergency about which to notify the primary health care provider. The history of diabetes would have been obtained on admission. Dark urine is expected in glomerulonephritis.

Which clinical manifestation in a client with pyelonephritis indicates that treatment has been effective? a. Decreased urine output b. Decreased white blood cells in urine c. Increased red blood cell count d. Increased urine specific gravity

b. Decreased white blood cells in urine Treatment has been effective when a client with pyelonephritis has a decreased presence of white blood cells in the urine. This indicates the eradication of infection.A decreased urine output, an increased red blood cell count, and increased urine specific gravity are not symptoms of pyelonephritis.

Which factor is an indicator for a diagnosis of hydronephrosis? a. History of nocturia b. History of urinary stones c. Recent weight loss d. Urinary incontinence

b. History of urinary stones A history of urinary stones is an indicator of hydronephrosis. Other causes of hydronephrosis or hydroureter include tumors, trauma, structural defects, and fibrosis.Nocturia is a key feature of polycystic kidney disease and pyelonephritis, but it is not associated with hydronephrosis. Recent weight loss and urinary incontinence may be factors in renal cell carcinoma but are not associated with hydronephrosis.

Which sign or symptom, when assessed in a client with chronic glomerulonephritis (GN), warrants a call to the primary health care provider? a. Mild proteinuria b. Third heart sound (S3) c. Serum potassium of 5.0 mEq/L (5.0 mmol/L) d. Itchy skin

b. Third heart sound (S3) When a third heart sound (S3) is heard in a client with chronic glomerulonephritis, the nurse needs to contact the primary health care provider. S3 indicates fluid overload secondary to failing kidneys. The primary health care provider would be notified and instructions obtained.Mild proteinuria is an expected finding in GN. A serum potassium of 5.0 mEq/L (5.0 mmol/L) reflects a normal value. Intervention would be needed for hyperkalemia. Although itchy skin may be present as kidney function declines, it is not a priority over fluid excess.

When assessing a client with acute pyelonephritis, which findings does the nurse anticipate will be present? (Select all that apply.) a. Suprapubic pain b. Vomiting c. Chills d. Dysuria e. Oliguria

b. Vomiting c. Chills d. Dysuria The findings the nurse expects to find in a client with acute pyelonephritis include: vomiting, chills, and dysuria. Nausea and vomiting and chills along with fever may occur. Dysuria (burning), urgency, and frequency can also occur.Suprapubic pain is indicative of cystitis, not kidney infection (pyelonephritis). Flank, back, or loin pain are symptoms of acute pyelonephritis. Oliguria is related to kidney impairment from severe or long-standing pyelonephritis.

The nurse is teaching a client the precautions to take while on warfarin therapy. Which client statement demonstrates that teaching has been effective? a) "I can use an electric razor or a regular razor." b) "When taking warfarin, I may notice some blood in my urine." c) "Eating foods like green beans won't interfere with my warfarin therapy." d) "If I notice I am bleeding a lot, I should stop taking warfarin right away."

c) "Eating foods like green beans won't interfere with my warfarin therapy." Rationale: Teaching about the precautions of warfarin has been effective when the client says "that eating foods like green beans won't interfere with my Coumadin therapy." Vitamin K is not found in foods such as green beans, so these foods will not interfere with the anticoagulant effects of Coumadin.Warfarin "thins" the blood, so the risk for cutting oneself and bleeding is very high with the use of a regular razor. The client needs to use an electric razor. Clients must apply pressure to bleeding wounds and must seek medical assistance immediately. While they may need to discontinue warfarin therapy, the priority is to apply pressure to the bleeding area and seek medical care. Blood in the urine of a client taking warfarin therapy is not a side effect. The client must notify the primary health care provider immediately if this occurs.

Which statement made by the nurse during an admission assessment for a client who is HIV positive demonstrates a nonjudgmental approach in discussing sexual practices and behaviors? a) "you must tell me all of your partners' names, so I can let them know about possibly being infected." b) "i hope you use condoms to protect your partners." c) "have you had sex with men or women or both?" d) "you don't participate in anal intercourse, do you?"

c) "have you had sex with men or women or both?"

Which point is most important for the nurse to include when teaching assistive personnel (AP) about protecting themselves from HIV exposure when caring for HIV-positive clients? a) "always wear a mask when entering an HIV-positive client's room." b) "talk to the employee health nurse about starting preexposure prophylaxis." c) "wear gloves when in contact with clients' mucous membranes or nonintact skin." d) "wear full protective gear when providing any care to HIV-positive clients."

c) "wear gloves when in contact with clients' mucous membranes or nonintact skin."

The nurse is preparing to obtain a sterile urine specimen from a client with a Foley catheter. What technique will the nurse use? a) Disconnect the Foley catheter from the drainage tube and collect urine directly from the Foley. b) Use a sterile syringe to withdraw urine from the urine collection bag. c) Clamp the tubing, attach a syringe to the specimen, and withdraw at least 5 mL of urine. d) Remove the existing catheter and obtain a sample during the process of inserting a new Foley.

c) Clamp the tubing, attach a syringe to the specimen, and withdraw at least 5 mL of urine.

A client with peripheral arterial disease (PAD) has a percutaneous vascular intervention. What is the priority nursing assessment? a) Dye allergy b) Gag reflex c) Pedal pulses d) Ankle-brachial index

c) Pedal pulses Rationale: After a client with PAD has had a percutaneous vascular intervention, it is essential for the nurse to assess for pedal pulses. Priority nursing care focuses on assessment for bleeding at the arterial puncture site and monitoring distal pulses to ensure adequate perfusion. Pulse checks must be assessed postprocedure to detect improvement (stronger pulses) or complications (diminished or absent pulses).Ankle-brachial index is a diagnostic study used to detect the presence of PAD. This is not necessary after percutaneous vascular intervention. It is imperative to assess for dye allergy before performing the procedure. Gag reflex is checked after procedures affecting the throat (e.g., endoscopy, bronchoscopy).

The nurse in the cardiology clinic is reviewing teaching provided at the client's last appointment regarding hypertension management. Which actions by the client indicate that teaching has been effective? (Select all that apply.) a) Reports walking the neighborhood once weekly. b) Reports eating fast food frequently to cut down on food costs. c) Weight loss of 3 lb (1.4 kg) since last seen in the clinic. d) Reports eating a low-sodium diet. e) Reports drinking one less cup of coffee daily.

c) Weight loss of 3 lb (1.4 kg) since last seen in the clinic. d) Reports eating a low-sodium diet. e) Reports drinking one less cup of coffee daily. Rationale: Teaching about hypertension has been effective when the nurse notes that the client has been on a low-sodium, diet has lost 3 lb (1.4 kg) since the last clinic visit, and has cut down on caffeine. Clients with hypertension need to consume low-sodium foods and would avoid adding salt. Weight loss can result in lower blood pressure. Caffeine promotes vasoconstriction, thereby elevating blood pressure.Although eating out may be cost-saving, fast food is often higher in saturated fat. The goal is to exercise three times and not once weekly.

Which concept is the highest priorityfor the nurse to consider in planning care for the client with HIV-III who has candidial stomatitis? a) cellular regulation b) gas exchange c) comfort d) nutrition

c) comfort

Which practices are generally recommended to prevent sexual transmission of HIV? (Select all that apply.) a) oral contraceptives taken consistently b) natural-membrane condoms for genital and anal intercourse c) latex gloves for finger or hand contact with the vagina or rectum d) latex dental dam genital and anal intercourse e) water-based lubricant with a latex condom f) latex or polyurethane condoms for genital and anal intercourse

c) latex gloves for finger or hand contact with the vagina or rectum d) latex dental dam genital and anal intercourse e) water-based lubricant with a latex condom f) latex or polyurethane condoms for genital and anal intercourse

A nurse is giving a group presentation on cancer prevention and factors that cause cancer. Which statement by a client indicates understanding the education provided? a. "Most cancer is hereditary." b. "Red meat helps to prevent cancer development." c. "Nearly 1/3 of cancers in the United States are related to tobacco use." d. "If I eat a healthy diet and exercise I will not develop cancer."

c. "Nearly 1/3 of cancers in the United States are related to tobacco use."

The nurse receives report on a client with hydronephrosis. Which laboratory study does the nurse monitor? a. Hemoglobin and hematocrit (H&H) b. White blood cell (WBC) count c. Blood urea nitrogen (BUN) and creatinine d. Lipid levels

c. Blood urea nitrogen (BUN) and creatinine In the client with hydronephrosis, the nurse monitors the client's BUN and creatinine. BUN and creatinine are kidney function tests. With back-pressure on the kidney, glomerular filtration is reduced or absent, resulting in permanent kidney damage. Hydronephrosis results from the backup of urine secondary to obstruction.H&H monitors for anemia and blood loss, while WBC count indicates infection. Elevated lipid levels are associated with nephrotic syndrome, not with obstruction and hydronephrosis.

After receiving change-of-shift report on the urology unit, which client does the nurse assess first? a. Client postradical nephrectomy whose temperature is 99.8°F (37.6°C) b. Client with glomerulonephritis who has cola-colored urine c. Client who was involved in a motor vehicle collision and has hematuria d. Client with nephrotic syndrome who has gained 2 kg since yesterday

c. Client who was involved in a motor vehicle collision and has hematuria After the change-of-shift report, the nurse first needs to assess the client who was involved in a motor vehicle collision. The nurse would be aware of the risk for kidney trauma after a motor vehicle crash. This client needs further assessment and evaluation to determine the extent of blood loss and the reason for the hematuria because hemorrhage can be life-threatening.Although slightly elevated, the low-grade fever of the client who is postradical nephrectomy is not life-threatening in the same way as a trauma victim with bleeding. Cola-colored urine is an expected finding in glomerulonephritis. Because of loss of albumin, fluid shifts and weight gain can be anticipated in a client with nephrotic syndrome.

A newly admitted client who is diabetic and has pyelonephritis and prescriptions for intravenous antibiotics, blood glucose monitoring every 2 hours, and insulin administration would be cared for by which staff member? a. RN whose other assignments include a client receiving chemotherapy for renal cell carcinoma b. RN who is caring for a client who just returned after having renal artery balloon angioplasty c. RN who has just completed preoperative teaching for a client who is scheduled for nephrectomy d. RN who is currently admitting a client with acute hypertension and possible renal artery stenosis

c. RN who has just completed preoperative teaching for a client who is scheduled for nephrectomy The client scheduled for nephrectomy is the most stable client. The RN caring for this client will have time to perform the frequent monitoring and interventions that are needed for the newly admitted client.The client receiving chemotherapy will require frequent monitoring by the RN. The client after angioplasty will require frequent vital sign assessment and observation for hemorrhage and arterial occlusion. The client with acute hypertension will need frequent monitoring and medication administration.

When caring for a client who had a nephrostomy tube inserted 4 hours ago, which is essential for the nurse to report to the primary health care provider? a. Dark pink-colored urine b. Small amount of urine leaking around the catheter c. Tube that has stopped draining d. Creatinine of 1.8 mg/dL (160 mcmol/L)

c. Tube that has stopped draining It is essential for the nurse to tell the primary health care provider when a nephrostomy tube that was inserted 4 hours ago does not drain. It could be obstructed or dislodged.Pink or red drainage is expected for 12 to 24 hours after insertion and would gradually clear. The nurse may reinforce the dressing around the catheter to address leaking urine. However, the primary health care provider must be notified if there is a large quantity of leaking drainage, which may indicate tube obstruction. A creatinine level of 1.8 mg/dL (160 mcmol/L) is expected in a client early after nephrostomy tube placement (due to the minor kidney damage that required the nephrostomy tube).

A client with hypertension is started on verapamil. What teaching will the nurse provide for this client? a) "Consume foods high in potassium." b) Monitor for muscle cramping." c) "Monitor for irregular pulse." d) "Avoid grapefruit juice."

d) "Avoid grapefruit juice." Rationale: The nurse teaches the client who is taking verapamil to avoid grapefruit juice. Grapefruit juice must be avoided with calcium channel blockers, such as verapamil, because it can enhance the action of the drug.Foods high in potassium would be encouraged for clients taking diuretics, not calcium channel blockers such as verapamil. Bradycardia, not irregular pulse, is a typical side effect of verapamil. Muscle cramping may occur with statins, not with calcium channel blockers.

When caring for a client with an abdominal aortic aneurysm (AAA), the nurse suspects dissection of the aneurysm when the client makes which statement? a) "I have a headache. May I have some acetaminophen?" b) "I have had hoarseness for a few weeks." c) I feel my heart beating in my abdominal area." d) "I just started to feel a pain in my belly and low back."

d) "I just started to feel a pain in my belly and low back." Rationale: The nurse suspects dissection of an AAA when the client says that "I just started to feel a tearing pain in my belly." Severe pain of sudden onset in the back or lower abdomen, which may radiate to the groin, buttocks, or legs, is indicative of impending rupture of AAA.The sensation of feeling the heartbeat in the abdomen is a symptom of AAA but not of dissection or rupture. Headache may be benign or indicative of cerebral aneurysm or increased intracranial pressure. Hoarseness, shortness of breath, and difficulty swallowing may be symptoms of thoracic aortic aneurysm.

Which client who has just arrived in the emergency department does the nurse assess as emergent and in need of immediate medical evaluation? a) A 64 year old with chronic venous ulcers who has a temperature of 100.1° F (37.8° C). b) A 60 year old with venous insufficiency who has new-onset right calf pain and tenderness. c) A 69 year old with a 40-pack-year cigarette history who is reporting foot numbness. d) A 70 year old with a history of diabetes who has "tearing" back pain and is diaphoretic.

d) A 70 year old with a history of diabetes who has "tearing" back pain and is diaphoretic. Rationale: The client who just arrived in the ED and needs immediate medical evaluation is the 70 year old with a history of diabetes who has "tearing" back pain and is diaphoretic. This client's history and clinical signs and symptoms suggest possible aortic dissection. The nurse will immediately assess the client's blood pressure and plan for IV antihypertensive therapy, rapid diagnostic testing, and possible transfer to surgery.The 64 year old is most stable and can be seen last. The 60 year old and the 69 year old would both be seen soon, but the 70-year-old client must be seen first.

Which client will the nurse encourage to consume 2 to 3 L of fluid each day? a) Client with heart failure b) Client with chronic kidney disease c) Client with complete bowel obstruction d) Client with hyperparathyroidism

d) Client with hyperparathyroidism

When a client with diabetes returns to the medical unit after a computed tomography (CT) scan with contrast dye, all of these interventions are prescribed. Which intervention will the nurse implement first? a) Administer captopril. b) Request a breakfast tray for the client. c) Administer lispro (Humalog) insulin, 10 units subcutaneously. d) Infuse 0.45% normal saline at 125 mL/hr.

d) Infuse 0.45% normal saline at 125 mL/hr.

The nurse assesses blood clots in a client's urinary catheter after a cystoscopy. What initial nursing intervention is appropriate? a) Administer heparin intravenously. b) Remove the urinary catheter. c) Irrigate the catheter with sterile saline. d) Notify the health care provider (HCP).

d) Notify the health care provider (HCP).

A client is receiving unfractionated heparin (UFH) by infusion. What laboratory data will the nurse report to the primary health care provider (PCP)? a) Hemoglobin 12.2 g/dL (122 mmol/L) b) White blood cells 11,000/mm3 (11 × 109/L) c) Partial thromboplastin time (PTT) 60 seconds d) Platelets 32,000/mm3 (32 × 109/L)

d) Platelets 32,000/mm3 (32 × 109/L) Rationale: When caring for a client receiving UFH, the nurse notifies the PCP of a platelet level of 32,000/mm3 (32 × 109/L). UFH can decrease platelet counts. The PCP must be notified if the platelet count is below 100,000 to 120,000/mm3 (100 to 120 × 109/L).A 60-second PTT reflects a therapeutic value within 1.5 to 2 times the normal value. Mild leukocytosis (increased white blood cells) may be expected with deep vein thrombosis. A hemoglobin of 12.2 g/dL (122 mmol/L) reflects a normal reading.

The nurse is caring for a client with peripheral arterial disease (PAD). Which symptom will the nurse anticipate? a) Decreased pain when legs are elevated b) Unilateral swelling of affected leg c) Pulse oximetry reading of 90% d) Reproducible leg pain with exercise

d) Reproducible leg pain with exercise Rationale: The symptom the nurse assesses the client with PAD is reproducible leg pain with exercise. Claudication (leg pain with ambulation due to ischemia) is reproducible in similar circumstances.Unilateral swelling is typical of venous problems such as deep vein thrombosis. With PAD, pain decreases with legs in the dependent position. Pulse oximetry readings reflect the amount of oxygen bound to hemoglobin. PAD results from atherosclerotic occlusion of peripheral arteries.

Which part of the HIV infection process is disrupted by the antiretroviral drug class of nucleoside reverse transcriptase inhibitors (NRTIs)? a) clipping the newly generated viral proteins into smaller functional pieces b) activating the viral enzyme "integrase" within the infected host's cells c) binding of the virus's gp120 protein to one of the CD4+ coreceptors d) forming counterfeit bases that prevent DNA synthesis and viral replication

d) forming counterfeit bases that prevent DNA synthesis and viral replication

The nurse is performing discharge teaching for a client after a nephrectomy for renal cell carcinoma. Which statement by the client indicates that teaching has been effective? a. "Because renal cell carcinoma usually affects both kidneys, I'll need to be watched closely." b. "I'll eventually require some type of renal replacement therapy." c. "I'll need to decrease my fluid intake to prevent stress to my remaining kidney." d. "My remaining kidney will provide me with normal kidney function now."

d. "My remaining kidney will provide me with normal kidney function now." Effective discharge teaching for a client after a nephrectomy for renal cell carcinoma is indicated when the client says, "my remaining kidney will provide me with normal kidney function now." After a nephrectomy, the second kidney is expected to eventually provide adequate kidney function, but this may take days or weeks.Renal cell carcinoma typically only affects one kidney. Renal replacement therapy is not the typical treatment for renal cell carcinoma. Fluids would be maintained to flush the remaining kidney.

During discharge teaching for a client with kidney disease, what does the nurse teach the client to do? a. "Drink 2 liters of fluid and urinate at the same time every day." b. "Eat breakfast and go to bed at the same time every day." c. "Check your blood sugar and do a urine dipstick test." d. "Weigh yourself and take your blood pressure."

d. "Weigh yourself and take your blood pressure." When discharging a client with kidney disease, the nurse needs to tell the client to "Weigh yourself and take your blood pressure." Regular weight assessment monitors fluid restriction control, while blood pressure control is necessary to reduce cardiovascular complications and slow the progression of kidney dysfunction.Fluid intake and urination, and breakfast time and bedtime, do not need to be at the same time each day. clients with diabetes, not kidney disease, would regularly check their blood sugar and perform a urine dipstick test.

Which staff member does the charge nurse assign to a client who has benign prostatic hyperplasia (BPH) and hydronephrosis and needs an indwelling catheter inserted? a. RN float nurse who has 10 years of experience with pediatric clients b. LPN/LVN who has worked in the hospital's kidney dialysis unit until recently c. RN without recent experience who has just completed an RN refresher course d. LPN/LVN with 5 years of experience in an outclient urology surgery center

d. LPN/LVN with 5 years of experience in an outclient urology surgery center The charge nurse assigns a LPN/LVN with 5 years' experience in an outclient urology surgery center to insert an indwelling catheter in a client with BPH and hydronephrosis. Catheterization of a client with an enlarged prostate, a skill within the scope of practice of the LPN/LVN, would be performed frequently in a urology center.The pediatric nurse would have little exposure to prostatic obstruction and adult catheterization. Dialysis clients do not typically have catheters inserted, so the LPN/LVN from the kidney dialysis unit would not be the best staff member to assign to the client. The nurse who has been out of practice for several years is not the best candidate to insert a catheter in a client with an enlarged prostate.

When caring for a client with hemorrhage secondary to kidney trauma, the nurse provides volume expansion. Which element does the nurse anticipate will be used? a. Fresh-frozen plasma b. Platelet infusions c. 5% dextrose in water d. Normal saline solution (NSS)

d. Normal saline solution (NSS) To provide volume expansion to a client with hemorrhage secondary to kidney trauma, the nurse expects that normal saline solution will be used. Isotonic solutions and crystalloid solutions are administered for volume expansion. 0.9% sodium chloride (NS) and 5% dextrose in 0.45% sodium chloride may also be given. Lactated Ringer's solution may be used if the client has no liver damage.Clotting factors, contained in fresh-frozen plasma, are given for bleeding, not for volume expansion. Platelet infusions are administered for deficiency of platelets. A solution hypotonic to the client's blood, 5% dextrose, is administered for nutrition or hypernatremia, not for volume expansion.

When preparing a client for nephrostomy tube insertion, it is essential for the nurse to monitor which factor before the procedure? a. Blood urea nitrogen (BUN) and creatinine b. Hemoglobin and hematocrit (H&H) c. Intake and output (I&O) d. Prothrombin time (PT) and international normalized ratio (INR)

d. Prothrombin time (PT) and international normalized ratio (INR) Before insertion of a nephrostomy tube into a client, it is essential for the nurse to monitor the client's PT and INR. The procedure will be cancelled or delayed if coagulopathy in the form of prolonged PT/INR exists because dangerous bleeding may result. Nephrostomy tubes are placed to prevent and treat kidney damage.Monitoring BUN and creatinine is important but is not essential before this procedure. H&H is monitored to detect anemia and blood loss. This would not occur before the procedure. This client would be on I&O during the entire hospitalization. I&O is not necessary only before the procedure, but throughout the admission.

Mass Casualty Triage Tag Colors & Meaning:

•Red: Emergent needing immediate attention (class I) •Yellow: Can wait short time for care (class II) •Green: Non-urgent or "walking wounded" (class III) •Black: Expected (and allowed) to die or are dead (class IV)


Set pelajaran terkait

Chapter 1: The Sociological Perspective

View Set

Chapter 6: Therapeutic Communication

View Set

SIE Security Industry Essentials (Sample Exam)

View Set

lesson 5-1 the legislative branch

View Set

Pt. & Special Needs Exam 1 Quiz Review

View Set